Ask Anna

Your questions answered here...

STAY CURIOUS!!!

This page is devoted to the daily questions our participants have asked....See responses and links below. 

If Anna doesn't know the answer... she will ask someone who knows!

Questions?? Summer Camp 2023

Q: Why where the drones so hard to make?

A[Hubert] The drones were a bit finicky to put together, but the real question I had was why did none of them get off the ground more than a centimeter or so? Turns out, there were a few things going on, some quite surprising. 

First, some drones went up just a little bit, but not higher. This we decided was due to the 'ground effect' where planes and birds have better lift when they fly close to the ground.  Next we looked at the battery pack. This had 4 1.5V AA batteries in series, for a 6V supply. So instead I used a 9V battery. This worked a bit better, but still not enough. Next we used a plug-in power supply with a knob to set the output voltage. This worked like a charm, with the drone easily flying to the top. However, it was going strong at 4-5V, less than 6. The difference was that the plug-in supply could deliver more current than our 6V battery pack. And indeed, using two battery packs in parallel made the drone go up several inches, but not to the top. Then we heard that the Hwaii girls had no trouble zipping their drones to the top! So what's the difference between Santa Fe and Hawaii? Air pressure (and density). Obviously, these kits were designed and tested at sea level, and not in the thin air of New Mexico.

A[Anna]:  “At heights with lower air pressure, the drone will find it harder to generate the lift required to fly. Because the air is less dense, the propellers have to work harder to force the necessary amount of air downwards to hover, resulting in increased battery consumption. Also keep in mind that air temperatures will be colder at higher altitudes, which will further affect your battery’s performance. “


Q: How can I better my understanding on physics 

A: [Andrea]  Take a physics class and ask lot's of questions.  Google search words you don't understand (e.g. quarks) and you will find videos and articles on them. 

Q: What is the likelyhood of robots taking peoples jobs? how might this effect the economy?

A[Ezra] This mostly depends on the type of job and the organizations control those jobs. Right now, we have a situation where delivery and manufacturing jobs are seen as replaceable by automated machines (think Amazon drones). I can't confidently predict what kind of jobs will be replaced or supplemented by robots in the future. In my opinion, the real meat of this question lies more on the corporations/organizations that stand to profit off of replacing labor. At least in the near future, a lot jobs in line to be replaced by automated machines/robots are managed by large corporations that can 1) afford the technological overhaul and 2) seek profit over the well-being of their employees. We are capable of implementing automation in an ethical way (i.e., supports workers' well-being), but I do not expect this to be the reality unless policies are put in place to specifically protect workers. In general, automation leads to lower consumer costs. Basically, if you remove humans from the equation, you don't need to pay salaries. Low prices in consumer goods tend to improve the economy as well. This is a massive question with a LOT of moving parts though. Tech advancements during the industrial revolution only really increased standards of living for middle and upper classes, and I think we will see a similar outcome if policies are not implemented along with increased reliance on automation. Note: I only focused on the goods+services industry because this is our present and near future situation- it is very hard to cover all jobs (e.g., nursing, research, education, etc.)

Q: What do neurons do?

A: [Emily] Neurons (and by that I mean biological neurons that we have in our brains) are physical structures that make up your brain (and they are found in other parts of your body too!) that are responsible for carrying messages throughout your brain and to the rest of your body. We can think about neurons as having two main parts: the part that receives messages from other neurons and the part that sends messages to other neurons. The receiver part gets messages from other neurons (this happens by the sending neuron releasing brain chemicals called neurotransmitters). If there is enough excitatory activity, the neuron will then send its own message using the sender part.

A “mathematical” neuron does basically the same thing. It takes in information from other “neurons” its connected to and adds them together with some basic scaling (e.g. one input might be multiplied by 2 whereas another is multiplied by 1/3). If the neuron that adds all these numbers together is above some kind of threshold (usually 0) it will send information to the next neuron.

Q: Does one necessarily have to be good at math to go into a science field?

A: [Ezra] In my experience, I did the most math in college. In my day-to-day work, I am not deriving equations, but my comfort with logical processes is paramount. I often struggled with exams in college (they were all STEM courses). I really wanted to be a scientist, so I worked to keep up by going to professors' office hours for extra help. Your professors will want you to succeed, and your innate skill in math is nothing compared to your passion to pursue science. You probably shouldn't hate doing math, though. Also, most professors do allow homework extensions and sometimes exam retakes (but don't slack off!) You're at college to grow and a good professor will help you do just that.

A [Emily] : I love this question so much! One does not necessarily need to be good at arithmetic to be a productive scientist. I cannot reliably multiply or add numbers on the spot and I have at least two calculator apps on my phone. What one should try to master is real math, and by that I mean mathematical thinking. There is so much more to math than arithmetic and algebra and these are the kinds of things you get to learn once you have a decent understanding of the basics (e.g. arithmetic and algebra). The ability to think through a problem, identify what is unknown and what is known, and distill things into patterns is an important part of science. I would actually wager that statistics is more important to science than math. But this kind of thinking isn’t restricted to the math classes. In reality, this kind of mathematical thinking (Ezra used the work logic) gets developed through basically every class you take, from physics to chemistry to psychology to English. Math is really much more of learning how to think than how to add.


Q: How does menace work mechanically?

A [Emily]: The sequence of events that MENACE uses is all the matchboxes are on a table top with marbles on them. When it is MENACE’s turn, someone goes and pulls the box that has the same board on the front and they might shake it up a bit. Then whichever marble ends up in the designated spot (we used the lower left corner, but it could be the right corner or some other designation as long as its consistent) will decide where MENACE is play next. This box is held off to the side until the game is over. If MENACE won, marbles are added to each box that chose a move. If MENACE lost, marbles are taken out of the boxes. Then you get to start over all together. This is an example of a machine learning system that does not require any mechanical manipulations or robotics whatsoever and in that way, I think it is beautiful in its simplicity.

Q: If in half lives atoms sometimes lose protons they become a different element, in those cases how is it posable to tell what that atom was and therefore how long its  life was/is.

A:[Jluie] Once an atom decays, it is impossible to tell if it was once another element.  Although if you have some radium-226, you would expect to find radon-222 (the daughter product of the alpha-decay, in the area.  Any radon in the vicinity is assumed to have decayed from the radium.  Each isotope has its own half-life.  I attached an image of the radium-226 decay chain and you can see the half-life of each isotope and it’s decay mechanism.  The eventual conclusion of the chain is lead (Pb)-206 a stable isotope.  Given enough time a hunk of radium-226 will become lead-206.

A[Thomas]: That’s correct! Atoms can change element through decay, both from the loss of protons (like in Alpha decay, the nucleus spits out 2 protons and 2 neutrons, decreasing the element number by 2) and from the ejection of electrons from the nucleus (like in Beta decay, a neutron spits out an electron and becomes a proton, increasing the element number by 1). For any given atom in nature, it is not possible to know for sure how it became what it is. Maybe it was formed in a star, maybe it was bombarded with radiation, maybe it decayed from something else. What we can do is isolate certain atoms and watch them decay over time and measure the path that those decays usually take (google “decay chain”). For example, we know from watching Uranium-238 that it will eventually decay into Lead-206. So if we find a rock with lots of Lead-206 and some Uranium-238, it is probably reasonable to assume that most of the Lead used to be Uranium. It’s especially suspicious if we find Lead in places where it shouldn’t be chemically, and it happens to be in a place that Uranium would fit fine. Sometimes we use the ratio of Lead and Uranium in that situation to determine how old something is. When the rock formed, all of that Lead must have been Uranium, and we can calculate how long it would take for the measured percentage of it to turn into Lead. That’s how old the rock is (google “U-Pb dating”).

Q: What is the difference between a nuclear physicist and nuclear engineer? Is there a difference?

A: [Julie] To me a nuclear physicist is more interested in the theory and experiments to prove the theory, while an engineer is more hands on, application of the theory.  But in reality either education will provide similar job opportunities.  I choose engineering because I just wanted to get through school as fast as possible, get a bachelors, and start making money.  But a higher degree in either field will lead to a more research based profession.

A[Laurie] A nuclear physicist is mostly concerned with how the nucleus behaves, either when left alone, or excited when it gets hit by a photon or another particle. This can be a very broad range of subjects. Understanding how a nucleus emits radiation can lead to developments in fields like medical treatments, energy generation, and spectroscopy for material identification, especially for threat reduction applications. In addition, there are certain strange behaviors of the nucleus; exceptions to widely accepted rules, which might lead us to revise our thinking about some aspects of theoretical physics.

A nuclear engineer is more concerned with applying this knowledge.  Most nuclear engineers I know work in the nuclear power industry. Normally you would think of reactors, but there are also increasing developments in small sources of energy. Particularly,  nuclear engineers are looking at using nuclear power in small engines for spaceflight rockets.

Q: what prupose to the straws serve on the drone?

A[Hubert]: There were two uses for the straws. One was to make four lightweight legs, so that the motors and the propellers were not sitting on the table. The other use was as a sleeve to run over the dowels to guide the drone.

Q: What flexibility is there in the creation of new technology? Such as copyright or global safety. Also, are there devices that can sense the creation or use of dangerous machines?

Q: just how can I get more time to work on projects

A:[Anna] you can use it on the breaks, lunch and you can come early or leave later. You could also take the project home and work on it if your parents are ok with it ;)

Q: What makes something raidioactive?

A: [Julie] An isotope is radioactive if the nucleus is “unhappy” (technical term is unstable) with the number of neutrons/protons or the configuration of those neutrons/protons.  This has to do with the nuclear forces within the nucleus and honestly I don’t really understand nuclear forces.  I’m sure someone understands nuclear forces, but it’s not me.  The net result is that the nucleus releases energy to become more “happy” aka stable.  This energy can be in the form of alpha (helium nuclei) particles, beta (electrons) particles, gamma rays, or neutrons.  If the nucleus is one of the really heavy elements (uranium or higher), there is an option for spontaneous fission, where the entire nucleus splits into two smaller nuclei and a few extra neutrons.

A: [Thomas] Remember that since protons are positively charged, they really want to get away from each other. Neutrons don’t have a charge but do have a different strong attraction to protons and other neutrons, so they kind of hold the nucleus of every atom together even though the protons would otherwise want to shoot away. This makes a delicate balance of very strong forces. If an atom has too many protons, too many neutrons, or is just overall too big, then the nucleus will be unstable and will start falling apart, producing radioactivity. Some radioactive materials are forged in huge explosions in space which are strong enough to assemble unstable atoms, and others are created when stable atoms are bombarded with radiation, making them unstable. For example if a Helium-3 atom gets hit with a neutron, the neutron will stick and the atom will kick out a proton (radiation). Now the atom is Hydrogen-3, but it is still unstable because it has too many neutrons. After a while, one of the neutrons will spit out an electron (more radiation) and become a proton, and the atom will go back to being Helium-3.

Q: When working with nuclear material how do you know when a reaction would lead to an explosion.

A: [Julie] Nuclear material in and of itself cannot “explode”.  A nuclear weapon is designed precisely with a number of materials, including fissionable material and high explosives to achieve a nuclear explosion. 

If you are referring to a criticality accident, where a large amount of radiation is released due to the chain reaction of one fission causing additional fissions and it’s repeated millions of times such that anyone standing near the accident would receive a lethal dose of radiation, we use computer simulations to determine if precise configurations remain subcritical.  There are general “rules of thumb” that we can use to give general guidance, such as we know that as long as one is handling less than 500 g of Pu-239 it will not go critical.  Also if one is handling solution, generally 6 liters can be handled at a time without it going critical, even if one had more than 500 g of Pu-239.  But for most configurations we use computer simulations.  If one is handling the material there usually is no indication until it is too late.  If a neutron counter is set up next the material as it is handled, one could see increased neutrons coming from the fissionable material as it gets closer to a critical state.  This is usually only done for experiments as the monitoring equipment would be in the way for the operation.

A[Thomas]: Careful math! Many factors contribute to criticality including amount of material, concentration of material, surroundings of the material, and what the material itself is made of. Even if criticality is achieved, it is almost impossible for the material itself to explode by accident. The main danger is release of radiation which is harmful to humans and the release of heat which itself can cause an explosion if the material is surrounded by something like water which will try to expand when it gets hot. That’s why Chernobyl exploded. It wasn’t a nuclear explosion – it was a steam explosion caused by really hot nuclear material.

Q: Has there recently been a big radioactivity mishap?

A: [Julie] The most recent large scale radioactivity mishap is the disaster at Fukushima Daiichi nuclear power plant in March of 2011.  It occurred when a large tsunami sent waves over the protective walls and caused a loss of power to the plant and damage to the backup diesel generators.  Due to the loss of power, they could not circulate water through the core of the reactor and the water heated up too much and turned the water into steam.  The steam rose in the containment building and pressurized the building until it ruptured and released radioactive material into the surrounding environment.

The most recent criticality accident was at a fuel reprocessing facility in Tokaimura Japan in September 1999 due to improper handling of liquid uranium fuel.  They were making fuel for a nuclear reactor.  The incident spanned approximately 20 hours and resulted in radiation exposure to 667 people and the death of two workers.

Another interesting radioactive incident occurred in Brazil, in September 1987 and is called the Goiânia accident.  An unsecured radiotherapy source was stolen from an abandoned hospital site. It was subsequently handled by many people, resulting in four deaths. About 112,000 people were examined for radioactive contamination and 249 of them were found to have been contaminated.  The radioactive material glowed and people put it on their face as makeup or played with it and one child even consumed some as it fell on their sandwich.

A[Thomas]: It depends on how we define big. In the nuclear world, we set the bar very low for what counts as a mishap because we want to address even very minor problems before they get big. I am not aware of anything recent that I would consider “big”, especially relative to the many disasters from other sources of energy like fossil fuels and hydroelectric which have both recently had very impactful accidents.

Q: In what ways is having such a powerful tool on our phone such as the app we were shown beneficial to us?

A[Hubert] For one thing, the benefit of this app is that you can do cool things like do demos in summer physics camps like ours. But people actually us many of these things, such as the magnetometer to locate things underground, the gps data to map where things are located, the light meter to help with photography, the sound meter  to measure noise levels etcetera.

Q: Why do wires have negative and positive sides?

A: [Andrea] They tell you what direction the current (electricity or electrons) is flowing so you can make sure the circuit pieces are going the right direction

A: [Hubert] There are electrical components that have a polarity. A battery has a positive and negative terminal. There are other parts where you have to pay attention to polarity. Remember the LEDs, which don't work if you put them in backwards, Others are capacitors and transistors. Some components don't care. A resistor works the same way no matter which way around you put them in. Wires are like that: you can turn them around, it will make no difference, they have no polarity.


Q: Is there anyway we can prove the computer wrong about climate change and maybe have little less heat warming then the computed guesses.

A: [Erika] As climate scientists, we constantly try to show and prove to each other how the climate model is wrong, then make it better through new equations.  The climate model will never capture everything that is happening in our Earth System, but the more tests and rigor we throw at it and into it,  we believe it can give us more clarity on future projections and scenarios of the climate.   There's a famous quote about computer models to keep in mind, "All models are wrong, but some are useful."  We don't know what the final climate sensitivity will be (final heating so so much GHG accumulation), so that is where a lot of work is being put to try to find a useful answer. 

Q: What is an example of light being refracted in nature/a natural setting?

A: [Hubert] The most obvious one is rainbows. Refraction occurs as sunlight enters a raindrop and again as it exits. Also, if you're looking at fish in a fishpond or a creek or an aquarium, the light is refracted, so that If you're a hunter and aim at where you see the fish, you will miss. Another natural setting is your own eye. The refraction of light by the rounded shape of the cornea at the front of your eye, plus the refraction in the lens of the eye further back, cause a sharp image to be focused on your retina, allowing you to see clearly. 

A[Laurie]: One of the most famous examples of refraction has to do with images coming out of water. Suppose you are standing on the bank of a lake, and you see a fish in the water. Because of refraction, the light from the fish bends when coming out of the water to your eye. This is because the water has a higher index of refraction than air. So, the fish isn’t along your straight line of sight.  It will appear higher up in the water than it actually is.  Fishing birds are experts in finding fish because they have learned to make adjustments for this effect.

Total internal reflection can also be found in nature and is responsible for desert mirages.  Also, think about what a fish sees if it looks up out of the water, especially at the angle of total internal reflection.


Q: If a laser is pointed long enough at human skin how long would it have to be to strt butning

Under clear skies, the energy of sunlight falling on a surface is about

1 kW (kilowatts) per square meter. So a panel that is 1 by 1 meters in size, pointed at the sun, receives about 1 kW of energy (though only a fraction of that  is converted into electrical energy). If I double the area of the panel, it can intercept twice as much energy, and I get twice as much energy out. My roof has 12 solar panels, and that is enough to provide all the electricity we use in the house.

Q: Why are laser lights so dangerous for the eyes?

Q: What makes engineering such a great career?

A: [Lauren] Engineering is a great career because it is always in demand as every industry requires engineering support. It is also versatile and there are many types of engineering that can be very different and there are many different types of jobs that you can do. It is a great bonus that it tends to pay very well compared to most other bachelor’s degree programs. However, it is only a great career if you enjoy engineering and the type of work you do. It is not for everybody, but on the upside, even if you were to go into engineering and not like one job, the jobs and industries available to engineers are so versatile and plentiful that it makes it possible to try different things with your degree and find what you enjoy.

Q: Why is absolute 0 the coldest something can be?

A: [Lauren] Temperature describes the average kinetic energy (energy that comes from motion) of molecules. Molecules are always vibrating, and the faster they are vibrating, the higher the temperature. Therefore, absolute zero is when the molecules do not vibrate at all and there is no motion. Since there cannot be a negative amount of motion, this is the coldest temperature possible.

Q: I was wondering why they haven’t repaired the giant magnet? And if that magnet can power all of New Mexico then why can’t they use all the energy they need?

A: [Andrea] The magnet could only power all of New Mexico for less than a second.


Q: What is a supernova in a very detailed description?

A[Ed]: Gravity is relentless and everything is pulling on everything. Jupiter and far away galaxies are pulling on you but the strength of gravity depends on distance so we only feel the Earth.  Gravity is always pulling thing together.  In principle, gravity would crush everything into something very small.   The force of gravity from one side of your head to the other side should, in principle, crush your head into something very small. Usually, something counteracts gravity.  In your head, it is the strength of the electrical forces within the molecules of your head which stops it from being crushed.  Same thing with the Earth: the Earth's gravity  does not cause the Earth to collapse into something very small because of the strength of molecules.  

The Sun doesn't collapse because there is a source of energy/heat which makes the atoms move randomly and fast (no molecules in the sun). The definition of temperature is how fast the atoms are moving. The heat causes the gas to have a pressure which counteracts the gravity.

Where does the energy come from?  If the sun was made out of a very efficient fuel such as coal or gasoline, it could only burn for about 7,000 years.  Energy is E = mc-squared, that is, mass (ie, stuff) is the same as energy (ie, light) and the sun converts mass into energy making enough energy to burn for 10 billion years.  Inside the sun, 4 protons and 2 electrons can come together and form a helium atom (nuclear fusion).  The helium atom weight less than the 4 protons and 2 electron and that difference in mass becomes energy (E=mc-squared) which means a hot gas which means a pressure to stop gravity.  Eventually a star runs out of atoms that can fuse together which means no more source of gas pressure to stop gravity.  It will collapse until some other pressure stops it.  If it is a small star like our sun, the electrons form a pressure because you can't pack electrons completely together (they repel each other).  If this electron pressure stops gravity you end up with a "white dwarf" star about the size of the Earth but weighing a million times more.  If the star is 10 times the size of our sun, gravity can produce such pressure that the electrons will merge with the protons forming neutrons and the whole star collapses to the size of Santa Fe (yet weighs a million times the Earth).  Effectively the whole star becomes one giant atom made of neutrons (a "neutron star").  When the process of electrons joining protons starts, it makes small particles call neutrinos which can easily go through the whole star within interacting.  They zip out of the star taking energy with them which reduces the pressure fighting gravity even more causing even more electrons to join protons causing the star to lose even more energy. The collapse, once it starts, goes faster and faster.  The whole star (a million times the size of the earth) can collapse in less than a second. As the inner most part of the star collapses to a neutron star, the outer parts of the star starts to collapse but  hit the inner stuff and bounces out.  So the inside of the star collapses (implodes)  while the outside explodes.   In a matter of seconds, the outside of the star explodes into a huge hot ball of gas.  That is a supernova (nova means new, it becomes a new star in the sky for about a month).  If one happens in our galaxy, they can be so bright that you can read a book at night by their light, so it is a super bright new star, a supernova. This happens every few hundreds of years. 

By the way, the big bang only made hydrogen (protons and electrons) and helium. All the carbon, nitrogen, iron, etc in your body was made inside a supernova and thrown back into space to eventually make a new generation of stars with planets and life-forms based on carbon (that is, us). We are made from the guts of supernova.  

If the star is more than 10 times the size of our sun, the gravity actually compresses the space itself (remember, completely empty space is actually something that can bend, vibrate, expand, compress, etc). The space compresses into another dimension and makes more space, actually an infinite amount of space even though that infinite amount of space is inside an area a few miles across.  It makes a black hole.  The inside of really big stars implode into black holes while the outside explodes into a supernova. 

A: [Andrea] there are 2 main types of supernova -- Type 1A and Core Collapse (Type II). Check out the wikipedia articles about them! 


Q: How does the size for the solar panel affect how much energy it produces?

Under clear skies, the energy of sunlight falling on a surface is about

1 kW (kilowatts) per square meter. So a panel that is 1 by 1 meters in size, pointed at the sun, receives about 1 kW of energy (though only a fraction of that  is converted into electrical energy). If I double the area of the panel, it can intercept twice as much energy, and I get twice as much energy out. My roof has 12 solar panels, and that is enough to provide all the electricity we use in the house.


Q: if the labs have stayed the same since theyve been built or if they have had new areas built on and around them.

Q: How do you make your resume sound professional

[Sara Mason] One important thing you can do is to make sure you get someone (a trusted adult, a teacher, your school counselor, or a smart and experienced friend) to proofread your resume for grammar, spelling, and mechanics (like punctuation). For better or for worse, problems in these areas can cause a bad impression in your resume that can be hard to overcome. It's also really important in my experience to get a second person to take a look for these things, because these 'little mistakes' can be very hard for you to see after you've spent so much time looking at a document. But another person can catch them immediately (all the more important for that person to not be the person reading your resume after you've applied for a job). 


The other important thing to do is to make sure your formatting has an easy-to-read, non-creative font (like Calibri), and that the sections and language used are "typical." Honestly, the best starting point is going to be the examples Cassandra provided. You can copy the same sections and format and you can even use some of the text as a starting point as long as you replace it with the information that's accurate about you. There is a lot of room for creativity and innovation in the professional world, but your resume is not the time to try to be really different. You want to stand out by presenting your achievements clearly, not because the format or font you use on your resume is really "weird" or hard to follow. 

[Lauren] It can be very helpful to look at example resumes online or use online resources for resume advice. Some basic advice that is always good for resumes is to keep formatting consistent (if you list titles and then dates then do that throughout the entire resume, etc), use proper language/avoid any slang, avoid any use of first or third person (“I” or “you”), do a grammar and spelling check, and have someone proofread. There are also templates you can find online to fill in to help make it look more professional. If you don’t have very much work experience yet, you can include sections for your relevant skills, volunteer experience, and awards or achievements. It also always a good idea to tailor your resume for whatever job you are submitting the resume for, so look at the job qualifications and try to write about your work/educational experience in a way that applies to those qualifications.

Q: Do you need to be involved with the military before college to eventually serve, or can you join later in life?

A: [Anthony] No you do not need to be involved with the military before college to serve. There are two different rank designations in the military enlisted and officer. Enlisted service members traditionally do not have a college degree to start with, but there are many programs available to earn a college degree while serving. Senior enlisted are highly encouraged to earn a degree to better advance their career. Officers are required to have a bachelor’s degree at the time of commission and are encouraged to earn a graduate degree to reach senior leadership. 

I would encourage anyone interested in serving in the military to reach out to their ROTC leadership at their high school, or college, and/or contact their local recruiter for more information. 

[Lauren] You do not need to be involved before college and can join later. However, one thing to consider is that people who have served will receive financial assistance from the military to pay for school, so there could be a great financial benefit to joining before college if you are sure you will want to serve at some point anyways.

Q: Is making cosmetics considered chemistry?

[Joan]  Absolutely!   Chemistry is about how different materials interact with each other.  Ordinary cooking in the kitchen is also chemistry.  Why does bread rise when you bake it?  It’s chemistry!

[Sara Mason] Lab Muffin Beauty Science is an awesome cosmetic chemist you can watch on YouTube. 

[Lauren] As with nearly any task, there is not only one type of job that is involved in achieving a goal. Different types of scientists, engineers, business people, and more would all work together in creating cosmetics. However, chemists and chemical engineers would definitely play a fundamental role in making cosmetics as the types of chemicals the cosmetics are made out of need to be produced to achieve their specific goal. I am sure there would also need to be biologists or biochemists involved to ensure the chemicals being produced are safe for humans. 


Q: How long does energy last in polymers that Sara described?

Q: What's the difference between being a scientist and an engineer?

[Joan]  I think of engineers as people who build useful things, like dams and wind turbines and giant magnets.  Scientists focus more on understanding the natural world around us.  What happens when two black holes collide?  The answer to that question probably will not impact my day-to-day life.   But the knowledge that scientists discover is then used by engineers to make the world a better place.  Albert Einstein discovered “general relativity”, that is, that mass “bends” space (remember the purple trampoline with the rock in the middle!).  That knowledge turns out to be essential when designing the GPS system that we all use for navigation 

[Lauren] Science and engineering often overlap, but the main difference in my experience is that scientists tend to work more with ideas while engineers work more with making the ideas possible and safe. In my experience working as an R&D engineer at LANL, much of my job has been to work alongside scientists to understand their ideas and what types of goals they are trying to achieve, and then use my engineering skills to assist in designing, testing, and troubleshooting the necessary components to make their ideas into reality. However, there are so many different types of jobs for both scientists and engineers that my generalization is not always the case.


Q: How do bath bombs benefit you while using them in a bath?

Q: What energy supplies prosthetics to work?

A: [Alex] Prosthetics are either passive or active. Passive prosthetics do not use any energy supply. Active prosthetics, for the most part, use battery power. There are some devices that are trying to harvest energy from walking. What that means is, your body creates and uses energy to control your muscles and there may be ways to harness, collect, and use that energy to power prosthetics. 


Q: How is biochemistry and astrophysics related?

A: [Hubert] One of the big questions we have is 'is there life out there?' Biologists are trying to figure out under which conditions life can exist (deep underground, or in places without oxygen, in hot springs, under the ice etc) and what makes organisms tick under extreme conditions, and what might be the minimum requirements for any form of life to exist.

Some astronomers are studying exoplanets (planets around other stars), or the moons of Jupiter and Saturn, to see if they can find signs of life there. They try to obtain spectra of distant atmospheres (in the case of exoplanets), to see if there are telltale gases in there (such as oxygen) pointing to life. Close to home,  there are plans to send robots to some of the moons of Jupiter and Saturn, and of course the robots on mars are working on this too. So the astrophysicists are talking to, and working with biologists.

Q: What other things does and electric security worker do?

Q: How do black holes form?

A: [Andrea] In a star there are two competing forces: gravity that pulls in and fusion that pushes out.  When the star dies one of these forces wins...if gravity wins it pulls the whole star into a small area creating a black hole

Q: What happens when two unstable isotopes come into contact with each other?

A: [ Aaron]:So the tricky thing about an unstable isotope is that, in many ways, it doesn't know that it is unstable.

When we think about unstable isotopes, we think of radioactive material.  If I have a milligram of something like 14C, that means that it has an activity of ~300 million decays/second, or ~8 mCi.  That's a lot.  But it's only because a milligram of 14C is over 10^19 atoms of 14C.  The vast majority of the atoms are just hanging out.

And when you talk about two unstable isotopes coming in contact with each other, it's a single atom of one isotope coming into contact with a single atom of another isotope.  They don't spontaneously explode/combust/annihilate/decay or anything else.  It's mostly just like a nuclear reaction with stable isotopes.  So they could just bounce off of each other, they could fuse together to form a different isotope.  They could exchange protons or neutrons to make products that are more or less stable than what you started with.  If they hit each other really hard, both could break apart.

The reason that this is so interesting scientifically is because our models for what will happen can say what the range of possible outcomes is, but they are not very good at predicting the likelihood of the different outcomes.  Further, they have been very difficult to study experimentally.  While it's really one atom reacting with one atom, most of the time the two atoms don't interact at all--It's VERY unlikely for a nuclear reaction to even take place.  To study reactions, you need a big target (think back to our milligram of 14C) and a LOT of particle trying to hit it.

A:[Lauren] There is not one easy answer to this question because it depends on the types of unstable isotopes. Different unstable isotopes undergo different forms of radioactive decay (alpha, beta, or gamma) as they progress towards becoming a stable isotope. Different types of isotopes (stable OR unstable) have different probabilities of absorbing (also called “cross section”) different types of radioactive decay, which would produce an additional radioactive decay. The probability of whether or not an interaction occurs will depend on several factors including the type of radioactive decay particle being emitted, and the isotopes present that could intercept that radiation.

Q: How big/strong of an impact (force) can strong magnets make when colliding from one another


Q: My blue light wasn't working today during coding and I tried switching everything around mulitpy times. Now I want to know what I was doing wrong and how I could have fixed it.


A. [Joan] Lots of reasons why things might not be working.  Flag me down sometime during a break and we can look at it.  Maybe the LED had its two legs (short vs long) switched?  Maybe the wire from the breadboard to the Arduino was connected to the wrong pin?  Programmers spend maybe 20% of their time writing code, and 80% of their time fixing errors in the code they already wrote!

Q: How do the lasers at concerts work? Is it the same with the way the light went through the shapes and are moved?

A:[Hubert] Your hunch is correct! The lasers at concerts are typically more powerful than the ones we have, and there is enough stage smoke to make the light beams visible. The beams are moved using mirrors and lenses much like the ones we used on Wednesday. Safety is still important, so the beams are pointed into the air, and not at the audience or the band.

Q: How are the solar panels on houses wired?

A: [Hubert] A solar panel produces a DC voltage (that is, there is a positive wire and a negative wire). The voltage is usually not that high.

The electricity in your house is AC (alternating current, which means the voltage switches from +- to -+ 50 times per second), and the AC voltage is 110V. To convert solar panel DC to house AC, you need a converter box. This has electronic circuits in it that can do this job. On my house, there are 12 solar panels wired in series (like you did on Wednesday), so that the voltage going into the converter box is 230 VDC. On many newer solar installation, there is a little converter on each individual panel, but they each do the same job, and it makes it easier to later add more panels.

Q: Is astrophysics hard to imagine how big space is

A: [Hubert] There is a famous quote about how big space is:

"Space is big. You just won't believe how vastly, hugely, mind bogglingly big it is" (from the Hitchhiker's Guide to the Universe by Douglas Adams). For scientists, it is still hard to imagine just how big space is, but we sort of get used to it.

Q. Andrea, why does HAWC use water?   Why can’t you simply put the sensors on the ground to measure the gamma rays?  Why water?  Why not some other medium (like oil, or helium gas, or some other substance)?

A: [Hubert] We want to catch these muons, and they are relatively rare. When one hits one of big water tanks, it produces a flash of light which is picked up by the phototubes inside. (btw, the tank is covered and the inside is pitch dark). Of course you could do away with the tanks and put the phototubes in little boxes on the ground, but the chance that a tube gets a direct hit instead of  a giant  tank is very much smaller. Going the other way, covering the whole mountaintop with tanks would be even better, but too expensive.


Next, why water? A muon traversing water will produce a flash of blue light. Clear (it has to be crystal clear) mineral oil would also work. But water is much cheaper to buy, and at the end some years from now, much easier to dispose of.ear


What about helium (or air, even cheaper)? When a muon comes through and produces a flash of light, the amount of light depends on the density of the medium, Helium (or air) is a gas with very low density, so would produce very very little light, so any gas is out. What about density higher than water? This would be solids like plexiglass, glass, or quartz. Too expensive and too hard to make so big.


So water tanks are the best, cheapest way to build a very large detector cosmic ray muon detector. 


A: [Andrea]. The gamma rays make a flash of blue light called "Cherenkov Radiation" in the water that we pick up with photo sensors called Photomultiplier Tubes. You can get Cherenkov radiation in other materials, but water is stable, cheap, and well understood. There is another experiment in China called LHAASO that also measures gamma rays. They use pieces of plastic call "scintillators" that also make a flash of light when the gamma ray hits them. So you're right! There are different ways to do this.  LHAASO has a higher energy threshold though because their scintillators are small and more sparsely spread out.  

Q: In computer programming see that it has to be exact and any little thing messes it up. What is the thing that messes up the programming the most?

A [Joan]: There are several things that can mess up programming.  One famous story is from the first Iraq war in 1991.  Saddam Hussein of Iraq invaded Kuwait.  The USA sent troops to push the Iraqis out of Kuwait.  We built a base for the soldiers in Dhahran, Saudi Arabia.  Iraq fired Scud missiles at the army base.   We defended the base by sending Patriot missiles to shoot down the incoming Scud missiles.  The Patriot missile system used radar to detect the incoming Scud missiles.  If the radar detected something, it would wait a brief time to see if the object detected moved in the sky to the expected location.  We didn’t want to fire a Patriot missile at a flock of birds!  One day the radar detected something (which was a Scud missile), and then tried to detect the object again 100 yards further in the sky.  The Patriot system did not detect the object a second time, so it did not fire the intercepting missile.  The Scud missile struck the army base and 28 soldiers were killed.  The Patriot system failed because it was looking at the wrong location in the sky to detect the object the second time.  The reason the Patriot system failed was because when it computed that location in the sky the computer had “numeric overflow”.  When the computer multiplies two numbers (such as 263 x 621 results in 163,323) it only has a limited amount of space to store the answer.  Scientists, such as the designers of the Patriot system, use really long numbers.  They might want to multiply the following two numbers, each containing 10 digits:

126,394.7589   x   512,187.2263

The answer should be

64,737,780,979.84823907

which is 19 digits long!  Really long numbers will not fit into the space that the computer has available to store a single number.  This was the problem that caused the Patriot system to fail to protect the soldiers in Saudi Arabia.

A [Hubert]: There are several levels of messing up. One is picky things like the missing semicolon at the end of a line of Arduino code. Things like that occur in all computer languages. But these are 'easy' problems, since the compiler which is trying to read your code recognizes that something is wrong, and tries to tell you where it thinks the problem is (missing semicolon on or before line xxx).

Good compilers are very good at spotting these picky errors and helping you to fix them.


But, once these errors are fixed, there is another level of messing up:

Much code is full of 'IF this, THEN do that' type of lines, where the programmer tries to carefully tell the computer how, for example, to make a peanut butter sandwich. Once you have thousands, or millions of lines of code, written by many different people, some of which was written 40 years ago, it becomes almost impossible to guarantee that your program will not mess up sometimes. Hackers try to spot these subtle logical errors in the code and make the code do things that were not intended by the authors.


Q: why is dark matter so important to the universe and how do we know it exists?

A: [Ed]We know dark matter exists because we something must be holding galaxies together. Galaxies spin around (and are beautiful) and we can tally up the number of stars in the galaxies from pictures of the galaxies. The galaxies are spinning so fast (our sun takes 250 million years to go around the milky way) that the stars would fly away. So there must be something matter providing more gravity that we don't see.  Since we don't see it we call if "dark"  (astrophysicists are not the cleverest of namers) and since it is a source of gravity it must be "matter", that is, like us, the earth, the moon, etc.  So we call it "dark matter".  Here is the embarrassing part: it makes up most of the matter in the universe and we don't know what it is.   We can just see 15% of the stuff making gravity  with our telescope (stars, planets, beautiful nebula, moons, gas, dust). The rest is dark matter. So it is real important, it makes up most of the universe. This is one of the biggest mysteries facing us: what makes up most of the universe?  Please become an astrophysicist and help us figure this out. 

We know black holes exist we see lots of different things that are very small but have some much mass they can only be a black hole. For example, we see stars that have a huge amount of mass (like 30 times our sun) but are only a few miles across. That can only be a black hole.  I say "see" because we can see them from the gravity waves they make.  We see something at the center of galaxies that is small (like the size of our solar system) but have enough gravity to rip apart stars.  It must be a black hole weighing a billion times our sun.  We see matter accelerated to near the speed of light yet flickers on the time scale of 1/100 of a second.  To flicker that fast it must be small, to accelerate blobs of gas the size of the Earth to near the speed of light it must have huge gravity.  It can only be a black hole.  

A: [Andrea]. I gave a public lecture on dark matter you should check out! https://www6.slac.stanford.edu/events/2015-11-17-cosmic-clue-dark-matter-mystery

Q: How are black holes investigated if they suck up everything?

A: [Ed] What light is made depends on the temperature of what is making it.  Really hot stars makes blue light, somewhat hot stars makes red light.  A black hole has two sources.  One is light from the black hole itself which is really really cold.  So cold that the light is actually in the form of radio waves.  The other source is the stuff that is falling into the blackhole.  That can be very very hot because the gravity of the blackhole is so strong it is ripping everything apart. That material can be so hot the light is x-rays. So basically, a black hole can make just about all forms of light from radio waves, to optical, to x-rays, to gamma-rays. 

Q: I still don’t understand dark matter, and I know we didn’t go over it a lot, but I don’t understand it’s purpose, or why we claim it exists when no one knows exactly what it is. 

A: [Hubert] (Hubert) When we look at a galaxy, we can count how many stars there are, and make a good estimate of how much mass there is in all those stars. We can also measure how fast stars in that galaxy move as they go around in their orbits around the galaxy center. Those two things should match: the gravitational pull of all the visible matter in the galaxy should be sufficient to hold the stars in their orbits and prevent them from flying off. Thing is, they don't match at all. The pull of all the stars is way too small to hold the galaxy together. Therefore there must be other stuff in the galaxy that we can't see, but somehow has enough mass to supply enough gravitational pull to hold the galaxy together. That invisible stuff is called dark matter, and all we know about it is that it is invisible, it has mass, and by carefully studying galaxies, we can tell how much there is and where it is. In fact, there should be some right where I'm sitting. It appears there is about five times as much dark matter as there is ordinary matter. We've been trying to figure out what this stuff is for more than 20 years now, with so far no success. 

A: [Anna] https://www.youtube.com/watch?v=7ctMWmmwIwg

Q: Can we work with arduinos more they are so fun?

A: [Anna] yes, the multi day projects starting this Friday (Robotic hand and Ohana) will make use of the arduinos.

Q: How does the electricity go through the copper tape?

A: [Anna] Copper is a metal. If you remember Priscilla, she mentioned that metallic bond is one where the electrons are shared between the atoms and they kind of flow between neighboring atoms (conduction band), holding the structure together and conducting electricity more easily. When electricity passes through metals, electrons carry electricity and spread it all over the metal. The mobility of electrons is the reason why metals conduct electricity.

https://www.youtube.com/watch?v=7RpyURBNZyU

A: [Hubert] Copper is a conductor (of electricity), All metals are conductors, as are other materials such as graphite (pencil lead), and water. In conductors, electrons are loosely bound to their atoms and you can push them around for example with a battery. In this case electrons are pushed in at one end of the conductor (copper tape), and inside all electrons move over and at the other end electrons come out. You can also push electrons around using changing magnetic fields, such as in the demo where a magnet is dropped down a copper tube.

Q: Will we be presenting or building and big projects?

A: [Anna] Yes, we will be building the big projects (Ohana, Robot hands) for few days and presenting them on the last day for fun!

Q: Why are black holes different sizes?

A:[Ed] It depends on how they were made.  Big stars, when they no longer have a source of energy to make the gas in the star hot enough to have a pressure to counteract the gravity of the star,  start to collapse which makes them denser, which makes them collapse more, which makes them denser, which make them collapse more, etc. etc.  The star compresses itself into a black hole.  To do that , it needs to be big enough to start the collapse process.  Only stars that are at least 3 times bigger than our sun can collapse into a black hole.  So the smallest black hole is several times bigger than our sun.  Black holes orbiting each other can merge together (making the gravity waves like we saw in the demo today) . So a bunch of black holes 10 times bigger than our sun can come together and make a black hole that is maybe 50 to 100 times the size of our sun.  But then there are those black holes at the center of galaxies which can be a billion times the size of our sun.  We are not sure if they were made in the big bang and were the seeds that galaxies form around or they grew that big by eating a lot of stars.  Bottom line: the smallest black hole is a few times the size of our sun and the biggest is billions. 

Q: if light could escape a black hole what might we see.

A[Ed]: What light is made depends on the temperature of what is making it.  Really hot stars makes blue light, somewhat hot stars makes red light.  A black hole has two sources.  One is light from the black hole itself which is really really cold.  So cold that the light is actually in the form of radio waves.  The other source is the stuff that is falling into the blackhole.  That can be very very hot because the gravity of the blackhole is so strong it is ripping everything apart. That material can be so hot the light is x-rays. So basically, a black hole can make just about all forms of light from radio waves, to optical, to x-rays, to gamma-rays.

Q: How is understanding the magnetic field relevant to saving energy/making energy more efficient?

A [Johana] Magnets are used everywhere! Magnets can be used to generate electricity, magnetic memory is the main way data is stored in a computer, and magnetic fields can even be used to levitate trains to make a very low friction train track. Just looking at the magnetic memory example—to read and write into magnetic memory has an energy cost. For any individual reading/writing operation the energy required is small, but they are so prevalent that computers and data storage account for nearly 10% of the world’s energy consumption. Understanding magnetic storage and making it just a tiny bit more efficient to read and write can therefore have a huge impact!

Q:Can you feel the Magnetic field around the Earth?

A[Hubert] Not really. The human body has no sensors that are sensitive to magnetic fields. The Earth's magnetic field is weak, but  even very strong magnetic fields cannot be felt. However, if you have something like a heart pacemaker, a strong magnetic field can upset it. Therefore there are lots of warning signs near strong magnetic fields.

A[Johana] We do not have a direct way to “feel” the Earth’s magnetic field. This is not just because the Earth’s magnetic field is very weak (<1 Gauss). We cannot tell a fridge magnet, which is roughly one hundred times stronger than the Earth’s magnetic field, is a magnet either. At least just by holding it by itself. If we hold a fridge magnet next to another magnet or close to the fridge, then its magnetic properties become apparent as we can feel the forces between the two magnets. This indirect way of investigating magnetic fields can also be applied to the Earth’s magnetic field, this is how a compass works!

Q: When generating electricity with magnets, does the size or type of magnet change the efficiency?

A [Johana]: Absolutely! If you remember our coil demo where we moved a magnet with respect to a coil of copper wire and measured the generated current: the stronger the magnet we use, the larger the current generated. The stronger magnet produces a greater maximum field, so the coil sees a much larger change in magnetic field when the magnet is moved. For the hand-crank generators this means more electrical current will be produced per turn of the crank.

Q: Would animals be affected visually by magnetic fields if they

A: [Johana] While some animals can sense magnetic fields, it is unlikely that they see the field with their eyes. While the mechanism is still not fully understood some birds have magnetite, a magnetic material, in their beaks. It is likely that this magnetite acts like a compass to help them navigate.


Q: How can smaller magnets be stronger than bigger magnets? 

A: [Anna] The strongest magnets are made of rare earth elements found on the periodic table. Neodymium magnets are the strongest rare earth magnets and the strongest magnets in the world. So if you use such magnets, you need a smaller piece of a magnet to get the same magnetic field as a magnet made out of other material. 

If you are talking about the electromagnets, an infinite electromagnet with a radius much smaller than the length, is only dependent on the current and the number of turns of the coil https://www.youtube.com/watch?v=8YWi-kUSOaI

However a finite length electromagnet the magnetic field depends on the radius so for the same current, the smaller the radius of the solenoid, the bigger the magnetic field in the center.

A [Johana] There is a material property called magnetization, which is a measure of the “amount of magnetism” in a material (more precisely, it is the density of the magnetic moment in a material). The magnetization depends on a variety of factors including the strength of the magnetic moment of each atom that forms the material, whether these moments like to align or anti-align with each other, and how closely packed the atoms are to each other. Materials with a large magnetization can be small, but very strong magnets while materials with small or no magnetization can be large, but very weak magnets (or even non-magnetic).

Q: What type of career field uses soldering ?

A: [Maria S] Electrical Engineering and technicians have to solder anywhere from a bit to a lot. It is essentially a career on its own depending of the requirements of the circuit. For low frequency circuits (like the bug pcb), a decent mastery will suffice, for high frequency (communication systems, like the ones in a smartphone or in the antenna towers), one has to be very good at it for things to work correctly. In the case of high-power components, the soldering also has to be very good (it requires specific material and the process might require holding the components with pressure). Space electronics require people with special certifications to withstand the conditions that the circuits will be subjected to! It is an entire profession of very skilled individuals!

Computer engineering, robotics, biomedical engineering (interaction with electronics) and electicians all of these can require soldering as well. There are even companies that focus exclusively on building PCBs!

A: [Hubert] In many fields of engineering or research, you have to deal with a lot of electronics. Much of it is commercial, and require no soldering, but often there are custom bits of electrical equipment, and most of the time some soldering is involved. Lots of soldering in my field of experimental particle physics.

Q: Quantum communications: How did they measure the photons? How does the photon detector work?

A: [Ray N] Great question! A single photon has very little energy, so we use detectors that have some sort of built-in amplification to make an output signal that's big enough to measure.  There are two types of devices we use for this. 

The simplest type is called an avalanche photodiode, and it's like a fancy solar panel. Just like a solar panel, it's made from a thin layer of semiconductor material, such as silicon or gallium arsenide. When the semiconductor absorbs a photon, the energy of the photon pushes an electron off a silicon atom, and that free electron (and the place it used to be, called a hole) make an electric current. That's how solar panels work. A single photon detector takes that same idea and adds a big voltage across the semiconductor, so that the one free electron accelerates and bumps into another silicon atom, knocking free a few more electrons, which then accelerate and knock off many more, and so on. The resulting avalanche of electrons makes a pulse of electric current which is big enough to measure. 

The other type is even more interesting: a superconducting nanowire detector. For these, we make a very small wire out of a special material (like tungsten silicide) and cool it down so much that it becomes superconducting (usually around 1 Kelvin). When a photon hits the wire, the energy of the photon heats up the wire so that it stops being a superconductor and we can detect the current. 

Questions from 2022 Summer Camp

Wednesday June 15

Q: Are we all going to present our robotic hands? Or is it only those that want to? Does our robotic hand have to be finished by Friday? Or can we continue to work on it after the camp is over?

(Anna) You can continue to work on it when the camp is over, but we hope you can show it to us on Friday!


Q: If we do decide to return the materials will we only have to return the pi-top computer and the arduino or will we have to also return other things?

(Anna) The website is now ready about what needs to eb returned and what doesn't need to be returned. https://sites.google.com/view/2022-summer-physics-camp/forms-and-end-of-camp?authuser=0#h.9eufldmc30t0


Q: I should have asked Dr. Daley about why the arch's of my feet hurt when I play basketball and why I think I have a pressure point in my knee even though I've never been seriously injured.

(Anna) I think the best would be for you to talk to your primary doctor who knows you best!


Q: What should we do if one of our servo motors are broken?

(Anna) Check out the Materials return process. We ask that you label broken or malfunctioning equipment when you return it so we can inventory it and not use it for next camp.https://sites.google.com/view/2022-summer-physics-camp/forms-and-end-of-camp?authuser=0#h.9eufldmc30t0


Q: I am very interested in knowing if I can either volunteer or shadow at Los Alamos as a high school senior. Please let me know if you know any information, it would be greatly appreciated 😄.

(Anna) To actually have access to LANL property and Laboratory there is a process that needs to happen. There are few instances when public tours happen and we are not allowed to bring anyone in that has not been through the access process approval. I strongly encourage you to apply for an internship at LANL or SNL. If you are still in High School, your point of contact would be Cassandra Casperson who you have met! https://www.lanl.gov/careers/career-options/student-internships/index.php


Q: In the experiment with straws we used water because it forms a meniscus do other liquids form a meniscus? Do any gasses form a meniscus?


Q: Would you please send me the recording of "Do You Want to Learn More About Working With Data?" from Saturday? I really wanted to attend the session but had prior commitments.

(Anna) I will look for it, edit the remaining and do my best to share it with you


Tuesday June 14

Q: What will be the side effects if people were not decontaminated after exiting the lab?


Q: Do any of the presenters offer some type of community service? (The presenter Sandra Begay got me thinking about that)

(Anna) Many presenters and Lab scientists volunteer in "community service" events. In fact the lab encourages staff to do it and offers up to 30 paid hours a year to lab employees to work in the community. I volunteer for the local shelters. When I didn't have kids, I used to cook a couple of times a month for the homeless shelter next to my house but when I had the kids, this was not possible for me anymore so, I started organizing collection events at work and collected food or clothes for the shelter. When I moved to Los Alamos, I offered to pick up items that people wanted to donate at people's houses and drive them to the right shelter (kids, or women.s or men's stuff). I am also a judge in science fairs and participate to other outreach events and started this camp 6 years ago. I am also a member of the J Robert Oppenheimer memorial committee (www.jromc.org) which fundraises and provides scholarships to high school students from certain schools and also organizes lectures and participates in science fair judging etc. At LANL I also organize technical seminars as part of being the chair of the P/T colloquium, participate in hiring panels and other project funding selection panels (LDRD) and in the past also organized the LANSCE Neutron School.

(Hubert) I do science presentations in Santa Fe schools: https://mrsciencesantafe.org/


Q: Can something become radioactive if it loses electrons as well?

(Hubert) Yes, there are radioactive decays where electrons are emitted from the nucleus. For historical reasons, these electrons are called beta rays. The nucleus that is left behind in many cases is radioactive. The other way a substance can lose electrons is by  ionization. In that case, the electrons do not come from the nucleus, but are stripped out of their orbits around the nucleus. The nucleus itself is not affected by this, so the substance does not become radioactive.


Friday June 10

Q: If you send a light into space that is perceived to not hit anything for a long time. will that light ever come back, or will it continue to travel and not come back? 

(Hubert) Imagine a bug walking on the (2-dimensional) surface of a sphere, trying to always walk in a straight line. It may seem to the bug that it is walking further and further from its starting point, but we know that it will come back to where it started from. If we put the bug on a flat plane (also 2-dimensional), it will never come back around. So whether it comes back or not depend on the shape of the 2-D space in which it can move. Similarly in our 3D world, whether you eventually come back or not depends on the shape of the 3D space of our universe. If it has a positive curvature, you'll eventually come back, if it is flat (or has a negative curvature) you won't.  


Q:If you do send something into space will it be affected by black matter?

(Hubert) Yes. That is because dark matter has mass, so it will pull on you, just like ordinary matter. When you go into space (or even when you don't), everything with mass is pulling on you. Most strongly the Earth, as it is close underfoot. The moon is pulling, the sun, pluto, even distant stars, though not much since they are far away, but still. When you're traveling in space, all these things, including dark matter, pull on you from different directions, affecting your trajectory.


Q: Why was 3d printing created?

(Hubert) Because it is sooo cool! Acually, there are many reasons. One is that you can print objects with shapes that you can not make any other way, like parts that are hollow, or sponge-like. ANother reason is speed: I use 3d printers because I can design a part on my laptop, and have it printed in a few hours. Before that, you would have to submit the design to a machine shop, and you would not have it delivered for weeks. It also can save material: in the past when for example you had a complicated metal part, you would start with a big solid block, and grind away at it until you had your finished part, plus a mountain of metal chips. Nowadays, you can even print houses with a big machine that squirts out a concrete slurry. 


Q: How did we discover we can create plutonium? Was it an accident?

No, no accident (more to come)


Q: Is there a way that students can come back to the camp next year to help?

(Anna) Students can apply to be "helpers"/"assistants" next year. This year Serina Martinez is an "assistant". We don't have a "rule" precluding students to reapply but we want to make sure as many students as possible get this opportunity

Thursday June 9

Q: Was the website Asia showed us the other day with the galaxies an example of machine learning?


Q: Does elevation have an effect on the bath bomb mixture?


Q: Is there anyone that went to NMT?

(Anna) I would have to check but we will have representatives from NMT next Friday come talk to y'all.


Q: Could there be a typo in the code about the conditional statement on page 17 step 33?

(Joan) NO.  there is no typo.

The conditional statement says:

if ( buttonState == HIGH )

Yes, we do want the double-= to check for equality.

if you wrote the following code:

if ( buttonState = HIGH )

you would still have legal code, but it means something else (something that you do not want).  The circuit would not behavior the way you wish and you would spend hours scratching your head trying to figure out this bug!

Wednesday June 8

Q: We have talked about how national labs focus on maintaining national security, so my question is "does that focus ever become corrupt from politicians?" If so how do scientists react and deal with that?

(Anna) There are different types of National Laboratories with different types of focus. https://www.energy.gov/national-laboratories. As we heard, scientists and National Laboratories do not make policies. We provide the information that the government then uses to base the policies. We report our findings in international conferences and in peer reviewed publications so others can learn from what we discovered or achieved and they can push the humanity's knowledge further. That is what we do day in and out. With COVID for example, scientists reported their knowledge at the time day in and out. As they learned more, they reported new information. Did they lied on the first paper? no!. They used the same scientific method you used in your science fair! At a given time, all our observations pointed to this conclusion. When we got more data, then the conclusion could change or still be the same but we remain true to the scientific method even when we don't like what we find out.


Q: How do solar panels collect energy from the sun?

(Anna) https://www.energy.gov/eere/solar/how-does-solar-work

"When the sun shines onto a solar panel, energy from the sunlight is absorbed by the PV cells in the panel. This energy creates electrical charges that move in response to an internal electrical field in the cell, causing electricity to flow."


Q: Will the world (or even the country) ever become completely solar?

(Anna) https://www.forbes.com/sites/davidrvetter/2021/10/11/could-rooftop-solar-really-provide-enough-electricity-for-the-entire-world/

"In their assessment published in Nature Communications, a team led by energy researchers at University College Cork in Ireland calculated a figure for the total surface area of all the rooftops in the world: some 0.2 million square kilometres—an area almost the size of the U.K. The authors then worked out that, if all the surface area was covered with solar photovoltaic panels, they could generate a total of 27 petawatt hours of electricity per year—more than the combined electricity consumption of the world in 2018.

That’s a lot of power. But the authors, led by Siddarth Joshi, a PhD student at University College Cork in Ireland, aren’t necessarily recommending that every rooftop on Earth must be festooned with panels. For one thing, the physical and logistical obstacles to such an operation would likely be insurmountable, and for another, the authors show that, from region to region, the costs of solar vary hugely. Just as crucially, electricity consumption could almost double in decades to come, according to McKinsey, dwarfing the total power consumption seen today.

But what the report does do is show how rooftop solar can best be deployed to help nations rapidly—and relatively cheaply—decarbonize and decentralize their power grids, as Siddarth Joshi himself explained"


(Hubert) In principle yes, but in practice it will be a combination of solar, wind and other sources. It is worth noting that a few weeks ago in the Netherlands, all electricity in the country was generated by solar and wind, and I believe the same happened last year in England, so it is not an idea that is far off. The key missing piece is energy storage to take care of night time and windless days.



Q: What was the scientific process that led to the invention of solar panels?

(Anna) I would say the photovoltaic effect discovery. 

https://www.smithsonianmag.com/sponsored/brief-history-solar-panels-180972006/

"It all began with Edmond Becquerel, a young physicist working in France, who in 1839 observed and discovered the photovoltaic effect— a process that produces a voltage or electric current when exposed to light or radiant energy. A few decades later, French mathematician Augustin Mouchot was inspired by the physicist’s work. He began registering patents for solar-powered engines in the 1860s. From France to the U.S., inventors were inspired by the patents of the mathematician and filed for patents on solar-powered devices as early as 1888."


Q: How many different branches of optics are there? And what are all the different careers under it?

(Marein)  Light can be used/investigated/controlled in such different contexts, that it is normally not useful to try to separate these into rigid “branches". If you are interested in Optics, you would normally study a general physics course, which would cover the basics of optics in the second year (once you have mastered mechanics, thermal physics, and electrodynamics). This will be quite an exciting process, and by the end maybe “optics” will mean something quite different to you. As with all of physics, the career possibilities are very broad - you can work as an optical engineer (e.g. to optimise machines that manufacture microchips), choose an academic career (in anything from astronomy to quantum optics), or even work as a patent lawyer or business consultant. You should not worry about finding getting a job, it’s more important that you identify a topic that fascinates you (easier said than done!)… I bet that the rest will fall into place.


Q: What do we do with the James Webb project when we are done? 

(Anna) Send us a picture of you and the project! Learn more about what it is expected to measure. Show it to your family and tell them what you learned!

(Hubert)... also, NASA wants your picture! https://webb.nasa.gov/content/features/educational/paperModel/paperModel.html

Tuesday June 7

Q: Did the big bang start from one point or multiple points or is that something that is still being researched?


Q: How many interns does LANL take at one time? During the summer do they take a select amount of interns, or is it just a simple apply any time and see if they accept?

(Anna) Here you can find LANL's external website about employees (which includes students and interns). And here you can find LANL internship website


Q: How long does is usually code a video game?

(Anna) Wikipedia says 3-5 years to complete the coding of a videogame- https://en.wikipedia.org/wiki/Video_game_development


Q: How do the resistors work? What happens without them?

(Anna) https://www.explainthatstuff.com/resistors.html


Q: Why did my white light bulb not work with the Arduino even when my code worked? (Maybe dead bulb?)

(Joan)  Maybe you can pull out that bulb and put in a different one?  We often need to do this kind of "Trouble shooting" to isolate and locate where the error is coming from.

(Hubert) You can test the LED using the pill battery from the copper tape activity: put the battery between the 2 legs, with the + side on the long leg. If the LED does not light up, it is dead (if so, throw it away and don't return it).


Q: Is there a way to ensure that you will get a job? I feel like getting a job comes with receiving a bunch of rejection and that's kind of hard for me so I just wanted to know if there is a way to make sure that the effort you put in something will actually pay off.

(Anna) Yes! put 100% in everything you do and someone will see your value. I personally believe the effort put into education is NEVER wasted. The more educated you are the more ahead you are from others and also the more you can offer to an employer and also the better pay you will get. It is always good to have mentors that guide your through choices. Today you heard Irene Qualters talk about subjects she thought were important to buid a career in computer science. So the same applies to all careers. I never imagined I would one day be a staff member at Los Alamos National Laboratory. I didn't even know where Los Alamos was!! At some point, while doing an internship while in college, I realized I enjoyed research and talked to people and they said that to be a researcher I had to do a PhD. So that's what I did! Then I wanted to get a job in Grenoble where I did my PhD and where my boyfriend was in France and i knew that to get a job there, I had to do a postdoc and apply for a job as a scientist so I came to the US to do a postdoc and... guess what... my boyfriend and I broke up and I asked at Los Alamos if they would hire me as a staff member and they did. And here I am 22 years later. Was it worth it??? I say yes!

Most of my friends from my Physics college, are working in finances or teaching or in their family business. Are they happy?? I hope so!

(Joan) Yes, getting lots of rejections is part of the journey, but you can't take it as a personal rejection.  It happens to everyone.  I recall the old rhyme about the farmer planting seeds in their field that goes "One for the rook and one for the crow, one for to rot and one for to grow".  Only 1 seed out of 4 actually grows, but if you plant enough seeds you get a fine harvest!.  I was terribly, terribly nervous when I first started to interview.  After each experience I felt a little bit less nervous for the net time!   So each interview did have a benefit.

(Sara) Rejection is incredibly hard for me, too. It’s very good advice to consider that even a job you don’t get is good practice going through the job application process. So rather than wasted effort, it is a learning opportunity. The more you can shift yourself towards seeing failures as chances to learn, the easier it can make it to get through them and come out stronger on the other side. Friends, family, and mental health professionals can also support you in having a strong sense of self worth that can survive when things don’t go your way.

(Marein) You will be successful if you find something that you love and then work hard at it. Please do not worry about rejection - you are way too young! Right now you should be more concerned about rejecting the wrong jobs (those that don’t really fulfil you). I would recommend focusing on "doing a great job" at whatever you choose to do. The good news is that this will be easier the more fun you are having. The tricky part just to find that thing that you really love. If you already know now (that’s rare), congratulations - this will give you a fantastic head start (but don’t be disappointed in case you find that things turn out different than you imagined). If you don’t know, just follow your intuition and explore different things for a bit. I want to assure you that your effort will pay off. Actually, I have often felt the opposite… you do have to put in a lot of effort (in college/academia), but often you get out much more than you put in.


Q: I wonder how coders make the coding software, there has to be some coding that goes into making those applications. Where did the first coding application get coded to make that application?

(Joan) Wow!  this is a GREAT question.  I could spend weeks to give you a proper answer!   The first program to ever run was on the EDSAC computer in England in 1951. How do we program a computer?  How do we give it instructions?   Everything (and I do mean everything!) inside the computer is just 0's and 1's, like:  0110100100011111100010101010100010001111111....  this is the only thing that a computer can understand.  Either the electric current is flowing or it is not.  The very first programmers had to write something like 10101000111110010001 in order to feed instructions into the machine (ack!  it was that bad! so hard to do).  When we write our program for the Arduino, then stuff that we type must be converted/translated into this sequence of 0's and 1's.  The program to do that is called a compiler.  Compilers were invented/written about 1960 and led to an explosion in the use of computers throughout society.


Q: How did the infrared sensor worked?

Inside, there is a semiconductor junction that produces a voltage when it absorbs a photon of infrared light, much like what happens in a solar power cell.


Q: What type of code are we currently using?

(Joan) We are programming in C++.  See Q/A below


Q: Are we allowed to keep anything after the camp? And if so what is allowed?

(Anna) At the end of the camp we will give you a list of the things that need to be returned and the things that you can keep (eg Book, calendar, magazine, notebook etc). Somethings like the potato or the M&M will be consumed while others like the optics boxes, tool boxes, solar panel box etc we ask that you return. Then there are the things that you can choose to keep in exchange of your stipend like the Pi-Top and Eleego box and Robotic hand materials (if you actually weren't part of this project team and want to do it on your own after the camp). You will get that list before the end of the camp, I promise!


Q: So we can get a recording of the class session if we need one?

(Anna) Yes, send email to spcyw-nm@lanl.gov and request the exact lesson.


Q: One question I have about today's lesson is what are the most commonly used programming languages right now, and do we have an estimate on how many will be created in the future?

(Joan) Currently the most popular languages are (in order): Python, Java, Javascript, C#, C++, PHP and R.   We are using C++.   There are thousands of different programming languages out there!  Just like there are lots of human languages.  Some languages (human and machine) are "dead", like Latin.  No one actually speaks them day-to-day any more.  We have no estimate at all about how many will be developed in the future!  Trying to predict the future is very difficult.  30 years ago I would have said that being able to talk to a computer and have it understand you would NEVER happen, it was too difficult to do.  But now we have Siri and Alexa!   Different programming languages send to be used a lot in a specific field.  For example, R is used extensively  in data analysis when a lot of statistics needs to be done (such as average, standard deviation, etc.)  Check out Sara Mason's demo about data on Saturday afternoon!

(Hubert) I don't know about the future, but here is a view of the past:https://www.researchgate.net/figure/A-brief-history-of-high-level-programming-languages-from-1956-to-2004_fig1_3248243. What the diagram does not show is that there are vast amounts of code in older languages (COBOL (1960), Fortran 77 (1977)) that are still in use today. Learning any of the modern languages will teach you how to 'code think', and after that you can pick up any other language easily, especially if they belong to the same family tree.


Q: How to code a led? (I struggled a little)


Q: How do you start from scratch when coding?

(Hubert) I find the best way is to have a project or a challenge (or in school, an assignment). There are countless 'xxx for beginners' and 'xxx tutorial' videos online. 


Q: Why isn't my blue light detecting my object? (IR Proximity Sensor)

(Hubert) email Hubert or Adriana and we'll help you troubleshoot it.


Q: What type of coding did we do?

(Hubert) It is sometimes called procedural: first do this, than that. If this thing happens, do that, if not, do the other thing, etc.


Q: In regards to building the James Webb telescope where do we find the instructions? Or are we meant to figure it out and assemble it ourselves?

(Hubert) There are no instructions but the picture in the first sheet shows the finished model. If you are not sure, here is a video: https://www.youtube.com/watch?v=fc9Fifyv8OY


Q: Can you code anything to the arduino?

(Hubert) Not anything - the onboard memory in the Arduino limits the size and complexity of what can run on it. Still, that leaves a large space to let your creativity run wild.


Q: Are the interviews on Saturday with Sara mandatory or optional?

(Anna) There are NO Interviews on Saturday. There are a number of activities (after 4:30 and on saturday) and challenges but none of them are mandatory. You chose to attend this camp and it is up to you to take advantage of the extra opportunities offered! Your call! For the stipend, attendance is required from 9:30-4:30PM


Monday June 6

Q: How would the battery and magnet experience change if you changed the scale or ratio of each?


Q: Are man made crystals and earth crystals truly the same thing?

(Hubert): In principle they could be the same thing, indistinguishable in any way. However, 'natural' crystals typically contain impurities, which can be specific to the place where they are found. For example, natural diamonds can come in different colors, depending on the (non-carbon) impurities. Of course, the folks that grow diamonds in the lab can add these too, leading to a cat-and-mouse game. The 'natural diamond' mining companies really don't want to lose their monopoly position.


Q: How long does it take to finish a PhD in astrophysics including all years of schooling?

(Asia) Great question, because this differs between the US and Europe. 

(Marein) Doing a PhD is one of the best parts of a science career, because you have great freedom in your research (and only few obligations). You must not see it as a “means to an end”!


Q: How long is the process of getting accepted into LANL?

(Anna) Well it depends! Are we talking about Internships? The important thing with internships is to make connections and approach mentors and tell them what you are interested in and make sure you submit all your materials early! I believe internship season starts in January and students should be placed by April.


Q: How come some people got graphite models and some got diamond rather than everyone getting the same?

(Anna) In fact some people got graphite, some got diamond and some got salt! Because these are the sets we purchased for the camp. I really like the graphite and diamond comparison because they are both JUST carbon and the way they bond with other carbons determines their crystal structure and their properties and that leads to the strongest material (diamond) and a weak material (graphite)


Q: Did anyone go to New Mexico Tech or know much about it? If so I would really enjoy talking to you and learning about what I can do to help me get into NMT and where it can take me into the future.

(Anna) We will have a representative from NMT the last day of the camp come talk about opportunities there but I am pretty sure there are a number of volunteers that attended NMT. Just ask!


Q: how does a telescope reach that ib space?

(Hubert): Not sure if this will answer the question, but here is a nice article about where JWST is in space:https://jwst.nasa.gov/content/about/orbit.html, and here is the launch and the first unfolding:https://www.youtube.com/watch?v=Whh0ffL9kKE


Q: How can you figure out the potential energy of the train car before you put it in the coil and the kinetic energy once it is in the coil?


Q: I was wondering what their opinions on choosing a career you like despite the income. I am a little worried about choosing a career I genuinely would like because I don't want to feel like I'm making less money than I could be.

(Hubert): The most important thing is to choose something that you like doing, since you'll be doing it for a long time. I also would not worry too much about money, since any career in a STEM field is likely to be a well-paying job

(Asia): Adding to Hubert's point - academic STEM careers usually pay less than their non-academic equivalents. It does not mean you cannot be a scientist and earn a good wage - it simply means that if you would like to be a professor one day, you should expect to wait for your high salary for a little longer than in private Research and Development setting.

(Joan): I have spent my entire career in Academia as a professor, and I have been paid pretty well.  I can live very comfortably and do the things I want to do.  Working in a University gives you FAR more freedom than working in Industry or even a National Lab.  No one punches a time clock, or fills out a weekly time card.  But you have to be careful to get a tenure-track position at a University and not a Lecturer position (which is strictly a teachinig job with little change for advancement or growth in your career)

(Sara) Everyone gets to decide what their priorities are when it comes to their job, and there are lots of priorities to consider aside from what you get to do in the job and how much money you make. Others include: where you get to live (is it a place you like? Is it a good place to have a family? Is housing affordable? Do other people from the same communities you belong to live there?), how stable the job is, how many opportunities for advancement there are, who you get to work with and for (this is really important! An awesome job is NOT AWESOME if you hate the people you have to be around), the degree of work-life balance, and how well the job meshes with your values and principles. For some people, making a certain income (beyond the minimum you need to survive) is a major priority, and that’s okay. Income was a big reason why I did not enter the job market as an academic linguist (which was my passion and what I really wanted to do with my life). The academic job market is very bad, with the number of people seeking university positions vastly outnumbering the number of positions available. I did not want to do a national job search, I wanted to continue to live with my partner, and that would mean I’d either be unemployed or making well below the poverty line if I only sought out opportunities in this field. My point is that it’s okay for you to prioritize doing what you love, making money, getting to live in a particular place, or any other thing that is important to you in your job search. I suggest you think about the kind of life that you want to have, and the most important criteria that a job needs to meet in order to support you having that kind of life. Your job does not have to be your only source of interest or satisfaction, but hopefully it can make a contribution to meeting your material, intellectual, and emotional needs along with other activities that you spend time doing.


Q: Is there a date set for when the first pictures/data from the JWST will be released?

(Hubert): July 12, see https://www.nasa.gov/feature/goddard/2022/first-images-from-nasa-s-webb-space-telescope-coming-soon


Q:Are the camp challenges mandatory? Like, could we just do one challenge instead of all of them?

(Anna) Challenges are not mandatory but you are here because you wanted this unique opportunity to learn and be surrounded by people that know more than most people you know and that wish you challenge yourself and inquire and be curious and that's why we set challenges. 


Q: For the tiny light bulb, is the shorter leg the positive or negative leg, vise versa.

(Hubert): The long leg connects to the positive voltage


Q:Could neutron scarleting be used in space on things such as the JWST to find signs of water and possible life in space? 

(Hubert): Neutron detectors were used to find water on the moon. But you need to be pretty close to do this, so it would not work for looking for water on exoplanets. JWST only looks at light, it does not have neutron detectors on board. But water leaves its fingerprints on the spectrum of light coming from distant planets, and that is how we look for water in space.


Q: Could growing crystals be used to learn about the history of the world by creating rocks and seeing how the same rocks were formed in a certain area? 

(Hubert): Yes, this is an active area of research. There are many questions having to do with the properties of materials deep inside the earth, and scientists subject crystals to enormous pressures to see what their properties are. These properties determine the flows of matter and heat deep inside the earth, and these in turn affect what happens on the surface, all through the earth's history.


Q: And why is copper so conductive?

(Hubert): I found this explanation pretty good: "We all know, that the smallest unit of all elements is an atom. An atom is a neutral particle with a positively charged nucleus at the center, and negatively charge electrons moving in a number of orbits around the nucleus. The electrical conductivity of an element is determined by its atomic structure. There are several millions of atoms in a piece of metal. Each atom of a metallic element has two or three electrons in its outermost orbit which are also known as valence electrons. The atoms form a metallic bond with each other to give a closely packed stable structure to the metal. During formation of these bonds, the valence electrons present in the outermost orbit are completely detached from their parent atom, and can move freely in the space that lies within the lattice structure of the metal. When there is no electric field, the electrons move around in a scattered manner in different directions. On application of an electric field, the electrons start flowing from one end to the other of the metal. Thus the large number of free-flowing electrons are responsible for conducting electricity through the metal. They act as charge carriers, and carry electricity through the structure of the metal.


Q: On a bigger scale using the magnets from the electromagnet demo, how do scientists stay safe but also complete their research if the magnets and materials they use are so incredibly dangerous? This also goes for every aspect of what we did today.

(Hubert): Many of us work with stuff that could be dangerous. These could be electrical hazards, chemicals, high-pressure gases, radiation, magnetic fields, sharp knives etcetera. When we work with these things, we put in place all kinds of protections, rules and checks, so that in the end when we show up for work we don't run the risk of getting hurt, as long as we follow the safety rules. Every year, we get lots and lots of training and reminders about how to work safely with all these things.


Q: Also I was wondering, how long does it take to find the one specific field you are interested in? Because usually when I think of chemistry I think of mixing liquid chemicals but now that I saw there is chemistry in solid materials/minerals, I was interested in how one discovers such specific and small aspects of lager fields.? Also how much of being in STEM provides uncertainty when it comes to finding a job or how much risk is involved in taking a job? Does that influence one's decisions?

(Joan) You learn such things bit by bit as you move through your education.  Each step along the road your studies become narrower and narrower.  In High School and the beginning of College you should take a wide range of course (Chemistry, Economics, Music, etc.).  The more classes you take the better you will know if Chemistry is really something you like.  And getting a Summer Internship (at LANL or Sandia or wherever) is a wonderful opportunity to see if you really like a field.

(Anna) I agree with Joan. I also think that the easier the degree... the more people take it and the less competitive we are. For me, I could have been a medical doctor, a teacher or a Physicist... somehow I ended up going to Physics. Was I thinking about research at National Laboratory? Certainly not! I had no clue!


Q: Can any two crystals be the exact same?

(Hubert): In principle yes.


Q: What does the process of getting a Doctorate degree look like?

(Joan Lucas) It took me about 5 years to get my PhD in Computer Science after I finished my Bachelor’s degree.  The first year or two is like being in college, where you take a bunch of courses.  While taking courses in CS, you get a better idea of which sub-field is more interesting to you, and you identify a Professor that you would like to work with.  If the professor let’s you join their team, you start working on that research project.  You do experiments, develop new algorithms, and push the envelope of human knowledge in the specific are of Computer Science.  Eventually you write your PhD thesis, which describes the particular project that you worked on, and the particular discoveries that you made.  A thesis is like a book.  My thesis was about 180 pages, and that was considered unusually short.  Bu the time you finish your thesis, you are the world’s expert in that particular, specialized topic!  For real!  After the first year or two, being a grad student in like holding a job.  You get paid to be a PhD student (unlike undergraduate study!)

Q: Does an electromagnet have a magnetic field or an electromagnetic field?

(Hubert): We sometimes use the terms interchangeably. You can think of a (non-moving) magnet as having an electromagnetic field with the electrical components all zero.


Q: Where can you find the materials for these types of experiments because I really enjoy doing them.

(Hubert) I like to devise experiments that you can put together with simple materials you (mostly) can find around the house or find at the hardware store: cardboard, clothespins, paperclips, wire, empty bottles etc. For some other experiments we know roughly what we want and buy stuff online.


Q: What are some things I can test my digital multi-meter on?

(Hubert): On the Voltage setting (look at the multimeter cheat sheet), measure the voltage of the different batteries in the kit: the AA batteries in the Doppler kit, the small 2032 pill battery from Monday, the 9V rectangular one from the IR sensor from yesterday, See what happens on different DC V settings. Also look at the potato battery activity https://www.mrsciencesantafe.org/summerschool/2020/potato.html

On the resistance setting, you can see that just about all metals have virtually zero resistance. If you look in the Elegoo kit, and get out the resistors, see how the resistance you measure compares with what is written on them: are 1K resistors all exactly 1000 Ohms, and are they all the same? Measure the resistance of graphite: find a pencil with the eraser missing, and measure the resistance from the back to the tip. Measure the resistance of paper, then of wet paper. How about water, salt water etc.


Q: How is the process from making a diamond and lead (graphite) different?

(Hubert): Graphite is basically made by cooking stuff that contains a lot of carbon, such as the tarry residue that is left over in oil refineries. This is done in many stages, but is not very difficult. Diamond in nature is formed when natural graphite is subjected to enormous pressures and temperatures deep inside the earth, and then comes up for example in volcanic processes. Industrial diamonds can also be made in a similar way, starting with graphite and applying high pressure and temperature. Another way industrial diamonds are made is by evaporating hydrocarbons in a near-vacuum chamber, and letting the atoms 'condense' into diamond. 


Q: How did the big bang happen and why? How do we know that the big bang happened and was there anything before it? 

See below


Q: What exactly are black holes?

Recall the picture from Asia's talk of a star, where the outer part of a star is 'held up' by the pressure of the radiation from the nuclear furnace in the center of the star? What happens when the furnace has used up all the fuel? It depends on the size of the star: if it is not very big, the star will somewhat gently settle into old age, getting dimmer and colder indefinitely. 

For bigger stars, remember the picture of an atom we saw: a very small nucleus, made from protons and neutrons, surrounded by electrons. This atom is mostly empty space: if an atom is scaled up to the size of a cathedral, the nucleus would be the size of a fly. When a big star collapses, matter in the center is crushed to where protons in the nucleus and electrons are crushed together into neutrons, and the empty space is squeezed out. The result is a neutron star, where one teaspoon weighs about 10 000 000 000 000 pounds. But what if even those neutrons get crushed? The matter gets denser and smaller, maybe indefinitely smaller, forming a black hole. Why this name?

This is why: for an object of a given mass, such as the Earth, the 'escape velocity' is the minimum speed a rocket needs to escape from the earth to faraway space. To escape from the Earth you need to go at least 11 km/second. But if you crush the Earth to 1/2 its current size, and you have to lift off from this smaller ball, you would need to go 44 km/s. Imagine you could crush the Earth to the size of a pea, then the escape velocity would be 300 000 km/s, equal to the speed of light. This means you could not lift off from this pea and reach space, and nothing could escape beyond this surface, not even light, so it would be perfectly black. What happens inside this surface (called the event horizon) can never be observed.


Q: What should we do with wires we coiled?

(Hubert): You can keep them and maybe you can use them in the future. Otherwise return them at the end.


Q: Why were there two kinds of black "atoms" in the mineral structure kit? (There was a 4 hole kind and a 5 hole kind)

(Hubert): These correspond to the different crystal forms that carbon can make. For diamond, 4 bonds in the shape of a tetrahedron, for graphite 3 bonds in a plane to make hexagons, and then 1 up and one down to stack the planes for a total of 5.


Q: Can you make more than one light turn on with a battery and copper tape?

(Hubert): Yes, look halfway down this page: https://www.mrsciencesantafe.org/summerschool/2021/house/index.html there are lots of different ways!


Q: Did the big bang start from one point or multiple points or is that something that is still being researched?

(Asia) We currently think that the Big Bang happened everywhere simultaneously. It is not an 'explosion' or any other event happening in space. It is a rapid explosion-like expansion of space itself. So the Big Bang happened all throughout space at the same time. The state of infinite density when the Universe was small beyond imagination is called a singularity and this initial singularity contained all of the mass and energy of the Universe that we observe right now. 

Sunday June 5

Q:  Are there any other STEM opportunities that you know of?

Yes, there are many in person and also virtual. Below are some that are FREE or nearly free:

(Asia) If you are interested in Astronomy/Astrophysics, here is a list of useful links for high school opportunities, including university-specific programmes like MIT or CALTECH: https://www.astrobetter.com/wiki/Summer+Internships

 

Q: Are there any open scholarships that you know of?

if you are asking applications that are open now, I don’t know but I suggest you look through the links below and any links others, smarter and more knowledgeable than me, might be able to add to:

Questions from 2021 Summer Camp

Friday June 18


Q: What does NM Tech have in terms of dual credit/underage admissions programs?

A: New Mexico Tech offers dual credit to students who attend high schools in the general area of the campus. Unfortunately we are not able to offer dual to everyone, but you are still always welcome to take summer courses with us. New Mexico Tech does also have a yearly summer program, The Summer STEM Experience, for rising juniors and seniors. We are looking to hopefully expand that program and open it up to all high school students in the coming years. You can always feel free to reach out to the Office of Admission for more information about that program. J. Cervantes

Q: What impact does volunteer hours have at UNM?

A: Volunteering can be very beneficial once you start to look for internship or job opportunities. Volunteering also enhances your academic career and helps you network with potential employers. Additionally, programs like the BA/MD or Nursing highly encourage volunteering for their application process. P. Nunez

Q: What impact does volunteer hours have at NMTech?

A: For admission, volunteer hours do not have any impact. We try to keep our admission process very straight forward which is why it is strictly based on GPA and test scores. Volunteer hours can help you out for other things though, such as non-institutional scholarships, future internships, on and off-campus jobs, etc. So do not feel like they are at all useless! J. Cervantes

Q: What impact does volunteer hours have at NMSU?

Q: On the material return form, what do you mean by "yes" and "no"?

Q: Athena - Where would the pocket nuclear reactors go to? And how long do these nuclear reactors last?

A: I am certain that they will be trained. However, similar to how spent nuclear fuel is transported, these microreactotors will be very safe and and their transportation will have to adhere to strict regulations to ensure everyone's safety. 

Q: Priscilla Nunez- Can you provide me with more information about the BA/MD program? I interested in pre-med so I would love to learn about the BA/MD program.

A: The BA/MD program allows high school seniors to apply to our Medical School. If a student is accepted into the BA/MD program they will have a reserve seat into our Medical School, will obtain a full ride scholarship for the undergraduate program, and will get mentorship. You can learn more about the program by visiting https://hsc.unm.edu/medicine/education/ba-md/. P. Nunez

Q: How old do we have to be for us to apply for an internship?

A: Most internships do not have an age criteria but might have a classification (sophomore, junior, etc.) requirement. I recommend looking for internship opportunities once you start at UNM. There are a lot of opportunities provided by our Career Services Office, https://career.unm.edu/.P. Nunez

A: At New Mexico Tech many of our students start internships the summer between their freshman and sophomore years. Because of this they are able to gain a great amount of experience during the entire course of their undergraduate degree! Many of these students are continuously asked back by the companies that they intern with every year as well, and many are even offered jobs by these companies at the end of their final summer before they graduate from New Mexico Tech. In the past I have even seen internship opportunities for high school students though, so don't be afraid to look around. You never know what kind of amazing opportunities that you can find! J. Cervantes


Thursday June 17

Q: when going into the military at the age you did, is it possible to do a different job that doesn't include the fighting?"

A: I had training to help me survive any situation (I was in the Marines after all), this said on my day to day job there was no fighting as I was an electronics tech. There are many other jobs one can do mostly using your brain and I can elaborate further (ie translator etc) . Also, the military does not just go to war but there are branches that do live saving missions; help citizens after a big flood/fire, medical emergencies, earthquake search and rescue etc etc Astrid Morreale

Q: Did being in the military benefit your educational journey? Besides financially.

A: Yes, of course! there is very little in this world that I think I cannot do, I also have a larger view on things than my peers . I am known for getting things done and efficient and that is probably thanks to my military training. I also traveled to Japan and learned Japanese and visited many many states within the USA. Today  I still have many friends that I keep in contact from the time I was in the military.-Astrid Morreale

Q: Did you do anything STEM related in the Marines?

A: Lots of STEM outreach, specially in racially segregated areas in the USA and with kids with disabilities in Japan. -Astrid Morreale

Q:  How was you're experience with the military? I was considering taking the same route as you, but I have some doubts that maybe you could clear up.

A: Well it depends what your ultimate goal is. I wanted to finance my education, gain financial independence  while getting a different view of the world along the way.- Astrid Morreale

Q: (Stacey Copp) Why doesn't pen ink dissolve like highlighters or color markers?

A: Great question! Anna did an experiment to best illustrate the answer. Here is a picture to show the answer. Basically, permanent markers and pens often have inks which don't dissolve in water (are not soluble in water), but certain organic solvents can dissolve the ink to enable chromatography. So if you need to clean sharpie off of your skin or clothes, maybe try hand sanitizer!

Q: Are there any national societies for asian-american students?

A; yes the most famous one is SASE: 

https://www.saseconnect.org/

Astrid Morreale

Q: Is there a paper that gives us information on what we get to decide what we get to keep or have at the end of our camp? Cause i need more info.

A: Look into the Materials Return google form sent on 6/17. If you follow the instructions, you will see what you can keep and what you need to return


Q: I would like to ask Alex Miera if she is worried that the research she does, especially any research going to the military/ the iron man, will go into unethical projects in the future.


Q: Why does the Sandia Hand only have 4 fingers?  

A: The pinky is actually unnecessary when picking up and gripping things.  Usually when you go to pick something up or move it, you naturally try to get your pinky out of the way when doing these tasks.  That’s why the Sandia hand only has 3 fingers and a thumb. C. Gilbert


Q: How old do we have to be to apply for internships? also is there any more programs like this that you know of?

A: To apply for the High School Internship Program at Los Alamos National Laboratory, you must at least 16 years old at the time the internship begins AND have senior standing in high school. The earliest a student can apply is the second semester of their junior year in high school, in order to intern the summer after completing their junior year, when they have senior standing. More information about the LANL High School Internship Program is available at https://www.lanl.gov/careers/career-options/student-internships/high-school/index.php. Information about addition opportunities are available on the STEAM Hub of Northern New Mexico website at https://www.nmsteamhub.com/


Q: I would like to ask Ms. Astrid Morreale more about her pathway in education. I was wondering if it was possible to join the military and the path she chose, but instead of engage in combat like she did and instead perform a STEM related service.

A:Not everything involves combat.  There are many other jobs one can do where you will be guaranteed to be off the front lines and be in a STEM field: bio-chemist, lab tech, medical field, engineering etc. Also there are branches that specialize in domestic live saving missions; help citizens after a big flood/fire, medical emergencies, earthquake search and rescue etc etc Astrid Morreale

Q: How would our society be now if the women pioneers of science didn't come forward and it remained exclusive to men?

A: What a really interesting question!  I don’t know that there is a specific answer, however, you can look around at what women do, what professions they are in and you see that they view the world from a different perspective than men. I think that if STEM were exclusively male than you would not see as many solutions to family and community problems.

Wednesday June 16


Q: We were more focused on the pros of exos. What about the cons of exos? Are there any issues with utilizing exos that later in life people have to deal with?

Q: How does biophysics help to study DNA?

I would say that the discovery of DNA not only helped us understand how genetic information is transmitted through generations; it also gave rise to a number of new fields, of which biophysics is one. Unlike pure biology, where experiments often involve observing live cells and understanding their peculiar behaviors without unnecessary physical manipulation, biophysics has a different approach and studies live cells from a physics perspective. For example, a biologist would ask a question— “what happens if a gene is no longer expressed, how does this affect the organism?” whereas a biophysicist would want to know if the fact that a given gene is not expressed (turned off) is affecting cell structure and other properties by say stretching it and recording associated forces and changes from a control case where a given gene is expressed (turned on).


You may find this Forbes article interesting— it also features the work Karissa did on the largest billion atom simulation of an entire gene. 


https://www.google.com/amp/s/www.forbes.com/sites/evaamsen/2019/04/25/what-does-dna-look-like-after-66-years-were-still-learning-more/amp/


There are many things we still do not know about DNA (for example, we study how the DNA folds to form chromosomes— I am attaching a video of a chromosome model I made below). There is plenty of work to do for the future generation of scientists like you! -Anna Lappala

Q: (Alex Miera) Will there ever be another type of exoskeleton in the future? Since you've already created a prosthetic hand - what other type of exoskeleton have you created?

Q: How does the hopper robot jump that high? Also how does it know when too jump and where to jump

A: The hopper drives around autonomously, but the jumping portion is controlled by a human and instructs the hopper how high to jump by setting the angle of actuators and force of spring mechanisms. C.Gilbert

Q: Question for Athena- Are the truck drivers that transport the nuclear power trained to work with it in case there is ever an emergency?

Q: I actually have a question about how the younger kids can possibly take advantage of the camps resources later in life.


Tuesday June 15

Q: If too much radiation causes cancer, why is radiation used in cancer treatments? Are different types of cancer more sensitive to radiation?

A: Great question!  All radiation is not equal. Radioisotopes used for medicine usually have very short half-lives (the time it takes for half of the isotope to decay away), usually hours to days.  This is important so there is the maximum impact on the diseased cells but effect on the body as a whole is minimized.  In addition, the delivery method for the isotope is essential.  Isotopes can be targeted by tethering them to a biological molecule like a protein that is attracted to the tumor, so the decay energy goes to a specific place.  The type of radioactive decay is also important.  Alpha emitters are good for small tumors (short path length) and beta emitters for larger tumors (slightly longer decay path).  All of these different aspects have to be considered and optimized when designing a radioisotope therapy. Eva Birnbaum

Q: Question for Heather- What was the most difficult part when you were apart of the launch team for perseverance?

Q: My question goes out to Marcey Hoover: You mentioned your passion to leave the environment a little better than you found it. I was wondering a few examples of what you do in your personal and professional life to fulfill this urge.

A: On a professional front, I take the time to speak at both internal and external events, encouraging people to take on careers that take action to combat climate change and contribute via science and engineering. Here is a link to a panel session that I moderated earlier this spring form the DOE YouTube channel:

https://www.youtube.com/watch?v=w7OA8MljFqw

On a personal front, we do simple things. For example, my family opts to take our car that gets better gas mileage on any driving trips to not use as much fossil fuels, we only heat and cool the portions of our house that are used constantly (e.g. kitchen and bedroom) to conserve energy, and we have planted low-water use landscaping at our house to conserve water!

Q: What do you think you will be working on in the coming years (as your job appears to "change throughout the project lifestyle")?

Q: I would like to know what factors were taken into consideration when developing the vaccine and how it would mimic the virus.

A: The main two considerations were safety and efficacy. 

The concept is as follows: the goal is to expose a person to a small piece of the COVID-19 virus enabling the person's immune system to specifically recognize the full COVID-19 virus; however this piece can't be too big (too much of the virus particle), otherwise this could result it infection. In addition, the synthetic piece of the virus needs to be stable enough to last in the body long enough to produce a robust immune response. The spike protein fits the bill perfectly. The spike protein resides on the outside of the virus and is the main piece responsible for interacting with our immune systems. The spike protein by itself connect cause an infection. It also is quite stable.


In terms of the Moderna and Pfizer mRNA vaccines, these vaccines consist of the mRNA surrounded by a shell. The shell and the mRNA need to be designed to be stable enough to survive the trip into our bodies' cells to the ribosomes, which will use the mRNA to make the spike proteins. Because the mRNA only codes for the spike protein, it cannot cause an infection. - Karissa Sanbonmatsu

Q: We’re all rovers tested in chambers or just some?

Q: For Amy Tainter- Do you think working so hard on the ranch growing up and in your adult life has helped you in your career? If so, in what way?

A: I can honestly say that most of my work ethics came from my two hard working parents (do not have an education) but worked so hard for my brothers and I to have everything we needed.  second came from raising animals where my dad taught me discipline, responsibility since 11 years old and dedication to my projects (steers, swine, lambs, and horses) year after year.  Then I have to thank my FFA advisor in High school who also disciplined, taught me all about life and first and for most to never be late and how to speak in front of a crowd.  Ranching is only a small fraction of hard work it takes dedication, perseverance and goals/drive to want it because it is not an easy life style.

Sometimes as a woman in any career whether it be ranching, Engineering, Physicists, chemist, nursing, doctor, teacher, or officer you might feel that you have to prove yourself at one point or another but at the end of the day, the only one you have to prove to is yourself that you give everything you do 100% and you love what you do!- Amy Tainter

Monday June 14

Q: What happens to the solar panels if it hails outside?

A: Part of designing and manufacturing any product is to build into the design features to protect it while it is operating.   This is what many engineers do in all types of industries including solar.  Additionally, governments (state and federal) can sometimes write laws that say products must survive in certain conditions: think buildings in earthquakes and hurricanes. IN which case, a team of engineers will design and execute tests to make sure their product meets the regulations. I was curious about hail on panels too because my family is going to have them put on our house and we get a lot of hail. So I found this information from the Dept of Energy https://www.energy.gov/eere/articles/hail-no-national-labs-solar-panels-survive-severe-storm Looks like we should be ok! M. Decroix


Q: Do you trust the vaccine even though it wasn't tested for 10-15 years, and it was tested on humans?

A: Yes I do because the science behind it is solid. very often when we start a project we do our homework thoroughly so there is a high chance of success. With Covid the pressure is a lot to get it right the first time  - there is no scope for mistakes and the learning curve by default had to be steep. A lot of brilliant minds and well-intentioned people were behind the vaccine. Also, remember that technology now allows to do a lot in a very short time. So it isn't like the old days where testing was much slower. Also, approvals take forever but in the case of the vaccine all hands were on board - we did not have just a couple of people at the FDA look at the application snail-pace but we had the technology scrutinized at multiple levels in parallel. Further, the mRNA technology used to make the vaccine has been in the pipeline since a couple of decades, scientists have been working on it for a long time but there was no immediate need for developing a new vaccine with this technology. Now the need is there and the technology was available - no reason not to use it. 


Hope that reassures you about this vaccine. -Rashi Iyer

Q: How can you trust the vaccine if there was no critical trials on it and it was only made in a couple of months and most shots/vaccines are made 10-15 years after they start working on it, and how do you know your not going to get sick from taking the vaccine, because the flu shot for example, people got the flu shot and they still got the flu. There was a guy he got the vaccine and a couple weeks later he got covid and he died.

A: Sandra Begay: Here's the information I trust to find out details about the COVID vaccine:: https://www.cdc.gov/coronavirus/2019-ncov/vaccines/keythingstoknow.html?s_cid=10493:cdc%20covid%20vaccine:sem.ga:p:RG:GM:gen:PTN:FY21


Q: Do you often face failures in your work, if so what are some examples, and how did you and your team overcome them?

A: LOL the work is wrought with challenges and failures. But the failures teach us a lot more than the successes - we have to trouble-shoot endlessly and we learn a lot during that process. We often have heated discussions about the correct approach and we don't always agree. The buck often ends with me and I have to make the final decision. I do so after much deliberation with everyone on the team - right from the undergraduate student who does a lot of the testing work to the senior scientist with the expertise. Often what we believe in theory should happen does not work out experimentally - so input from the students and postdocs is critical in the decision making. For eg., it was my idea to use hollow fibers for the lung but my graduate student and postdoc who grew the cells in the fiber and then performed the testing could not get it to work. we kept at it for nearly 6 months while we tried other alternative approaches. Finally, I pulled the plug on it. We make small and big decisions everyday that impacts the outcome of our work. I have been very fortunate that we have a great working team with few egos and a lot of ideas. In the end we all have the same goal. Rashi Iyer


Q: Are there gonna be more help sessions for the rover?

Yes! absolutely. Let's say from 5:30-7:30 for the next two evenings. That will give you a break and you can go get some dinner. Also the mentors usually meet about 4:30 to discuss how the sessions went. I may need to send a separate google meet link - let me talk to the other mentors and see how we're going to arrange a room and I'll post that information here and also send it out to the email list. - Matt

Q: Could we code from our own personal computers or do we need to use the Pi Top?

You  can use your own laptop, it is probably recommended. Here's how to download the Arduino IDE onto your laptop.

Go to: http://chamisaelementary.com/phys_camp/home.html

and you will see links for the Arduino IDE for Windows and Mac. The Windows link takes you to the App store and installs it. The Mac link puts a file in your Downloads directory and when you click on it, it should install. (I don't know Macs, but I think that is right).

Installing the QTRSensors library is exactly the same as on the Pi Top.

-Matt

Q: What is the best advice Rita Gonzales would give to incoming internship students when it comes to succeeding but also taking care of themselves?


Friday June 11

Q: How do I secure my network?

Answer by Caren Shiozaki:

1. I always change default login passwords and user names (these are what manufacturers assign so devices are ready to go "out of the box")

2. When I get a new device, I remove the bloatware (software and apps that the manufacturer automatically installs, but not that useful).

3. Strengthen security on the wireless router:  use strongest encryption protocol available; as in (1) change the router's default admin password; 

change the default SSID; disable WPS (wi-fi protected setup)... which allows a wireless device to join a network w/o entering the network password;

keep firmware on my router up-to-date.

4. I keep operating system updates/patches up to date on my devices.

5. I backup my data regularly.  I have a local separate device that is not networked, and I also have a backup in the cloud.

6. I run up-to-date anti virus software.

7. I use a VPN.

8. I use a password manager. This makes it easy to follow best practices of not reusing passwords, and having them sufficiently complex.

9. I do not use any digital assistants (like Alexa); I won't install RING, I won't use NEST.  I have a basic alarm system; if I wanted one with visual capability, 

I would configure my own using servers and webcams I control.

10. The only social media I am active on is LinkedIn, and even then I am very careful about what I post.  I don't use FB, Twitter, etc.  A lot of this stems

from the work I've had to do with the FBI and other government agencies, and because I am called to testify or be deposed in litigation.


Q: How do you think solar panels can benefit not only the planet but humans?

A: You could say that benefitting the planet is benefitting humans. 😊 But there are other benefits for reducing our use of carbon fuels such as reductions in rates of skin cancer and fewer strong storms that damage our homes.  Solar isn’t the answer for every place on the planet, but it is really great alternative source of energy for places like New Mexico. The increasing number of alternative energy sources is really exciting for engineers because now we can match the energy source to the need and location instead of always trying to pipe in fossil fuels.M. Decroix


Q: How much solar energy is needed to light up and entire neighborhood?

A:The amount of energy needed is determined by the number of energy consuming “things” in the neighborhood. Here is a neat little info sheet for street lights. https://hypertextbook.com/facts/2004/MarinaAvetisyan.shtml

If we assume every block has 10 street lights, then the amount of energy used is 10 times the energy of one street light (let’s assume 300Watts) so 3000 Watts for every block.  If your neighborhood is 10 blocks, then the street lights use 30,000 Watts. It adds up very quickly. The invention of large LED lights was a huge advancement because they require much less energy, ~100 Watts, so the same number of lights uses only 1/3 as much energy. BTW, this energy can be supplied by carbon fuels or solar or other forms of energy production. M. Decroix


Q:Ms. Anna, I was wondering if you could go over what will be included with the technology package when we choose it instead of the $300 stipend.

A: All information is on website

Q: How long does it take to destroy plutonium?

A: This question of the disposition of nuclear materials like Pu is one of the legacies that will pass to your generation, to your grandchildren, and to many generations beyond us.  

 

Keep in mind that radioactive materials can be safely handled and stored if respected.  They are not a death ray or a plague.  You are radioactive right now, and if you had a banana for breakfast your radioactivity probably doubled this morning (potassium).  My father-in-law survived the bombing of Hiroshima and lived until just this year, aged 93 years, when Covid caught up with him. Alan Hurd

 

Q: How can you build a professional relationship with people when you are still in the introvert stage?

A: As one introvert to another, I can offer a few of tips that have worked for me.

Thursday June 10

Q: Do you think we would still have to code for computers in the future?

A. Absolutely!  Remember the PBJ sandwich demonstration!  Computers can do amazing things.  Computers can learn new things on their own.  But they still need guidance from humans.  Perhaps in the future we will be better able to program using voice commands, rather than typing.  But programming and coding are not going away.  Joan

Q: When might the next rover be sent to space? What will be it's job and when do you think it might be launched?

Q: Why would computer programming be important to learn at our age?

A: Computer programming teaches problem solving and breaking problems into logical steps. This skill is extremely useful even if you do not end up programming as part of your job. Also many jobs require technology and coding skills - is it truly a superpower. In my view the trick is to find something you are passionate and see how coding can help solving problems - many times coding just for the sake of coding is 'boring' but when solving real problems it drives you. - Thomas

A. Computers and coding are everywhere in our world, so coding is a very valuable skill to learn.  And lhe younger you are when you learn, the easier it is.  Toddlers can learn a new language (perhaps Russian) very easily and grow up bi-lingual.  Learning a new language in High School takes years are effort (and your accent will never be as good as that toddler’s!   Even Anna still has an accent). - Joan

Q: Question for Joan Lucas and Thomas Proffen: What first made you interested in pursuing a career and education in coding/computer science/programming?

A: Thomas - funny enough this happened much later. I have a PhD in Physics and in the olden days, any code we needed to analyze data we collected the PhD students and later postdoc wrote themselves. I developed an interest in scientific software writing code, being admired for my coding superpowers :) Outreach started with science outreach and it was not until my daughter was in Middle School and participated in a online computer science competition that I go into coding and technology education specifically for girls and founded Oak Ridge Computer Science Girls and that led to meeting all of you this afternoon and have a wonderful day :)

A. Joan - I took a programming course in High School as an elective, and really enjoyed it.  It was a fun challenge to see if I could make the computer do what I wanted.  It is like solving a puzzle or playing a game.  Then I took another course, and then another…. and here I am!   I never was able to answer the question “where do you want to be in ten years?”.   I think life rarely is something that you can carefully plan out.  You just follow your passion, keep your mind open to new experiences, and see what happens!

Wednesday June 9

Q: Why are wavelengths so important?

A: The wavelength of light tells you how much energy a photon has. There’s a famous equation:

Energy = h times c/wavelength  = hc/λ

where:

E is photon energy (Joules),

λ is the photon's wavelength (meters),

c is the speed of light in vacuum - 3x108 meters per second

h is the Planck constant - 6.62606957 × 10−34 (m2kgs−1)

Remember, the definition of Energy is ‘the ability to do work. The unit of energy is a ‘Joule’, which means the work done by a force of one ‘newton’ acting through a distance of one meter.  One ‘newton’ is equal to the force that that gives 1 kilogram of material an acceleration of 1 meter/second/second.

All these unit definitions can be confusing, but just remember, light can do work, whether it is moving the vanes in a Crook’s radiometer, or heating up a solar panel to create electricity. The wavelength of light can ultimately tell us how much work that light can do.

Another reason wavelengths are so important is that they tell us what kind of light we are dealing with. For example, wavelengths from about 400 to 750 nanometers are in the range of visible light. X-rays have wavelengths from about 0.01 to 10 nanometers.


Q: Ms. Casperson, on one of your resume slides you had mentioned that it’s best to not use relatives and references on a resume, but would it be okay to list a relative that was a past employer? Or is it best to avoid listing relatives (past employer or not) all together?

A: It is best to avoid listing a relative as a reference because their reference will typically be viewed as biased in favor of the family member. That said, you should still list the employment in your resume and you can certainly discuss the position in your interview. 

Q: Ms. Lebak, I’m not sure if you mentioned it during the lunch break, but are there any internship opportunities at N3B? If not are there any volunteer opportunities?

Q: If someone did have a non-professional resume, what is somethings that would still help them qualify for the opportunity?  A: Lots of activities other than jobs can still give you relevant experience for a job posting. Just a few examples: internships, volunteering, participation in extracurricular school activities and clubs (like theater, Best Buddies, the GSA, etc.), participation in sports while still maintaining a high GPA, advanced coursework (AP, IB, etc.). Indeed.com has some suggestions for writing a resume with no experience here. - Sara M.

Q: With cybersecurity, how can we block scam callers on our phones?

A: There’s 2 ways. The best is to sign up for the National Do Not Call Registry https://www.donotcall.gov/   Also a lot of devices have the ability to manually block phone numbers.-Olivia Stella


Q:What are the requirements to become an ESA astronaut? What classes should someone take in college to become an astronaut?

A: The requirements are surprisingly simple! You need to: 


As for classes in college, essentially anything that ultimately leads to a degree in STEM - chemistry, physics, engineering or mathematics. You can also become a doctor and then be sent to space to investigate *in situ* the response of human body to 0G environment! 


Q: How does the milky way galaxy account for all of its planets and star without crashing and dying out?

Even though there are about 200 billion to a trillion stars in our Milky Way, the empty space between the stars in humongous. For example, from here to the next closest star is about 20 trillion miles. So space is vast and mostly empty. Here is an example: the Andromeda galaxy is about the same size and shape as our own galaxy. It is on its way here, and will collide with our own galaxy in a billion years or so. You would think that there would be a lot of collisions between stars. But because space is so vast, there will probably not be a single crash. 

(Hubert)

Another thing to think about is angular momentum. As you will learn in physics classes both linear and angular momentum are conserved in the Universe. In case you were wondering whether stars e.g. move towards the galactic centre and collide because of gravity, then this is not the case. A star stays in its orbit at a distance away from the galactic centre owing to angular momentum and does not change its course unless its host galaxy crashes with another galaxy, causing the star to gravitationally interact with other objects.


Q: What makes light exactly? Like how are the electrons in light seen by us?

Light is made by charged particles like electrons.  Electrons surround the nucleus of an atom in what we call ‘energy levels’, but they don’t always stay in one level. Sometimes the atom is disturbed and an electron can move to a more energetic level. The electron wants to get down to its original less energy level, and to do so it emits a photon of light which carries the extra energy away.  That is where most light comes from.

There are no electrons in the light that we see every day, just photons. Electrons do move around outside an atom (like an electric current), but they are very easily stopped when they hit something, even something as light as air. So they really never get to your eye.  Astronauts in space report flashes of light from electrons and other charged particles hitting their eyes coming from the vacuum of space where there is nothing to stop them. But we don’t see this at this surface of the earth.


The answer to the second part ( how is light seen by us): The light-sensitive layer in the back of your eye is called the retina. In the retina there are light-sensitive cells. In these cells are molecules that are sensitive to photons of particular range of color (or wavelength), or to just light of any color. When a photon of light falls on these molecules, the energy of the photon is absorbed by this molecule, and this causes a whole chain reaction of chemical and electrical activity, with the result that an electrical signal is fired off from the retinal cell towards the brain. In the center of the retina there are such cells that are sensitive to 3 different colors: mostly red, mostly green and mostly blue. Cells that are sensitive to light, but not to any particular color, are sprinkled over all of the retina. 



Q: What if you apply for more then 1 job, do you need to have a different reference or the same person ?

I have used the same person as a reference for different job applications, as long as that person can speak to my qualifications for that particular job. Also, it is good to let them know each time you use them as a reference . - Sara M.

Tuesday June 8

Q: Are we going to be doing mini project like every day?

A: On the first week, we are learning the concepts and doing small parts but next week every afternoon will be dedicated to working on the specific projects (MARS Robot or Ohana Hmestead)

Q: Do men feel threaten by women in science? Is it because the social norm at that time.

A: Remember that men and women are just human beings and we all have our own personalities and behaviors. Some are based in our genes and some come from our experiences. Some men will be threatened by new interactions and certainly there being more women in the workplace is new to some of them.  But I have also found some women can be threatened as well. So gender stereotypes really don’t take you very far when you are on a team trying to do a job. Interactions/Relationships with people are one of the hardest parts of any profession/job. And I think, on average, more women are aware of those difficulties than most men. I try really hard to look at everyone I interact with as someone to learn about. I don’t always succeed, but I try.    -Michele Decroix

Q: What does the alligator clips have that also helps light up the LED?

A: Alligator clips are made of metal, which conducts electricity, and they have a spring which makes it easy to clip onto other wires or electrical bits.

If you don't have wires with alligator clips, you have to strip the plastic insulation off some wire and wrap it around the LED legs, which is less convenient and more finicky.


Q: Do you believe Project Y would have as successful as it was without the women who contributed?

A: A: I think that the varied contributions of women were essential to making Project Y a success. The wartime laboratory at Los Alamos relied on women of science including medical staff, physicists, chemists, and mathematicians, but also needed the support of women to build a community for Project Y staff in what was then remote New Mexico. Women, like Dorothy McKibbin and Charlotte Serber, were in charge of important administrative offices in Santa Fe and in Los Alamos. Women in the military (the WACs) stepped in to take over previously male-only wartime roles, working as drivers and supply clerks. Local women from the Pueblos and nearby Hispanic villages worked in the technical areas and in laboratories, where their fine motor skills were essential in the assembly of small-scale experimental devices that led to the development of the Fat Man weapon. Women, like Frances Dunne and Norma Gross, made valuable contributions while working in hazardous field conditions. Without a doubt, women were key to the success of Project Y and to ending World War II, along with women working across the country at other Manhattan Project sites, such as Oak Ridge, TN; Hanford, WA; and New York’s Columbia University.-Ellen McGehee

A: Fun fact: There were no women engineers on Project Y. Engineering was not a common thing for women to do and still remains less common then women in science. Frances Dunne and Norma Gross did engineering work but they had science degrees. There was one woman, Miriam White Campbell, who joined the Women’s Army Corp (WACs) and worked as a draftsman for the project. She drew the final drawings to assemble the Little Boy weapon. Her work was engineering. Today we would call her a designer and she would probably have a mechanical engineering degree or a mechanical technologist credential. If she were working today, she would probably be doing a lot of 3d printing! There is an interview on You Tube where she talks about bringing her dog  to live with her in Los Alamos.   https://www.youtube.com/watch?v=R5Q0RP3x5pc -Michele


Q: How exactly does the copper tape work.

Materials that allow electricity to flow through it are called conductors. In this group are all metals, like copper, iron, aluminum etc. Also water, and all wet things, such as your body, potatoes, lemons etc conduct electricity.

Materials that do not allow electricity to pass are called insulators. In this class are plastic, rubber, cloth, paper, dry wood, air etc.

This is why for example extension cords are copper on the inside, and plastic on the outside. You can touch the cord and not get hurt, and why you don't want to touch the prongs of a plug while it is mostly plugged in.

Our copper tape is made of metal, which is a conductor, so electricity flows through it, while the paper we tape it down on does not. (Our particular copper tape has glue on it that is also made to conduct). So it is like using copper wires with a plastic insulation on it.

In the experiments we do in the camp, voltages are low enough that they are safe: you can touch the wires safely. The limit for this kind of safe work is 50 Volts. (the highest voltage we use is 9V)

(Hubert)


Q: Olivia stella: if you link multiple social media accounts, like linking spotify to instagram and instagram to apple, or something like that, does it increase the risk of you getting hacked?

A: Yes, because if they get your apple password, they have access to everything else that uses the apple password. And even if it’s just social media accounts, they might be able to use the information to access more sensitive accounts (shopping accounts, bank, etc.) Here’s a good article from 2014 (yes, it’s a persistent problem)  - https://www.pcmag.com/news/reusing-passwords-across-social-media-sites-dont-do-that


Q: What makes the motor we made today with the copper wire,battery, magnets and paper clips spin ?

Remember that an electrical current running through a wire produces a magnetic field. In our motor we made, this effect stronger by winding the same wire around 7 or 8 times - this makes the magnetic field stronger. The next thing to remember is that magnets can push or pull on each other. In the little motor we made, the parts are arranged such that the magnet at the bottom pushes and pulls on the wire-coil-magnet (when we send a current through) to make it spin.


Q: I forgot what the 3 things you need to make electricity was called so my main question is what were the 3 components needed to make electricity?

If the question refers to the potato battery, this is what you need: first you need 2 different metals, plus some fluid in between, like the juice in the potato, or orange juice, or even salt water.


Q: Olivia Stella: discussing about cybersecurity " will having similar passwords on multiple platforms increase the chances of being hacked?" This seems like an important thing to know to safeguard ourselves. Other than the lessons though, I was wondering if we would have a lecturer talk to us about medicine as I currently want to study medicine and it would be really cool to learn/explore pathways for it.

A: Yes, because if they get your apple password, they have access to everything else that uses the apple password. And even if it’s just social media accounts, they might be able to use the information to access more sensitive accounts (shopping accounts, bank, etc.). Here’s a good article from 2014 (yes, it’s a persistent problem)  - https://www.pcmag.com/news/reusing-passwords-across-social-media-sites-dont-do-that- Olivia Stella

Monday June 7


Q: Why are women so underestimated in the science and engineering field?

A: Underestimated is an interesting word because it depends on your perspective.  I think that the field of engineering has been so male for so long that it has just taken on the behaviors and thought processes of men. It is so normal now that no one really even thinks much about how it could be different.  Right now the profession is only about 20% female at the entry level so we are having a tough time breaking through with our thought processes. Areas of engineering that are changing fastest are Bio-engineering and energy/environmental because they have a higher number of women in the fields.  My area of Mechanical and Aerospace is still less than 15% female.  I do feel underestimated  sometimes but I am not sure it is a conscious thing most of the time. I believe I just solve problems differently and it seems abnormal to my colleagues.  It is frustrating!  It has been helpful for me to raise the consciousness of my male colleagues about how they solve a problem. I ask a lot of questions.😊 BTW, this is true in reverse as well. My mom was a nurse which is traditionally a female profession. When male nurses started being more “normal” there were a lot of the same discussions in that field. So it is really is about being the minority in a field and not always gender.-Michele Decroix

A: Because people are weak, and it has always been easier to settle for a stereotype (as in gender roles and racism) than making

 up our own mind. Unfortunately, these views can trickle down the centuries if they are not being questioned. But don’t assume that you will have a bad experience in STEM! There are strong efforts all over the world to change this situation.-Marein Rahn

Q: Why do some materials work better with different colors of led lights?

A: Every material in a circuit has a property called impedance and the value for impedance represents how easy or hard it is for electricity to “flow” through the material. Copper has a pretty low impedance and conducts electricity well. Other conductors have higher impedance and conduct electricity poorly.  So if you use a different material with a different impedance in the same circuit, then you get different voltage and current output and you may not be able to light the same LED. Electrical engineers must choose materials carefully to get the right performance from circuits without “wasting” a lot of the energy, because the electricity that is not conducted through a material is lost energy. The “lost” energy is stored in the material which causes it to heat up and the components of the circuit can get very hot. Many electrical components that handle high power (like transformers in the electrical grid) need to be cooled, usually with oil. There is a lot of research happening in materials to develop broadly useable superconductor materials that can handle high power with very little loss in heat. This is a tough problem because the materials also need to be mechanically strong to withstand weather and tension between the poles and other design constraints. But if we can develop the right materials, then we can reduce the losses in the national electrical grid. This will allow us to move electricity around the nation further and faster without wasting energy.

Q: How do you get to be so determined about what you want to do so early?

A: In my case, I found something I was interested in at around age 14 (learning languages) and just kept looking for ways to keep doing that, eventually ending up with a PhD in Spanish linguistics. I never stopped liking it and still find it fascinating. However, it is not common to find what you want to major in and work on professionally when you are 14. If you aren't sure about that, there's no reason to feel bad or like you're falling behind. Also, even if you find what you want to major in and work on that early, it can still change later on (like it did for me). It is normal to have several different careers over the course of your life. - Sara Mason

A: I was not determined about what I wanted to do when I was your age. For some reason I wanted to be different than my sister. She liked science but was more interested in the medicine/biology side. I liked physics because I had to think through the problem. It wasn’t just memorizing things. Also, I could make things happen in the real world, like throwing a ball or shooting a projectile, and see the physical laws in action (gravity).  My sister and I took physics class together and I was better at then she was. That was very frustrating for her.  A little competition is healthy I think.  😊  My dad was a skilled tradesman in construction and worked with a lot of engineers. He said, “You should be an engineer”. I said ok but I did not know what it was. I just knew it paid well.  I almost dropped out my sophomore year in college because it was really hard. But I stuck it out and the next semester I took a class in thermodynamics and found my passion for studying fluids. From there I went on to work on designing airplanes and doing research in combustion. Life opens lots of doors. The trick is sticking around so you can walk through them when you are ready.-Michele Decroix

A: I think the biggest challenge nowadays is that there is so much stuff that catches our attention at the same time, so it gets very difficult to focus on one thing. The problem is that to develop a passion for something, you really have to get into it first. So my advice would be just to select one thing (really, anything. On this page alone I see several questions that could ignite and support a whole career) that you feel you care about, and start digging into it. Read a book, sign up for a course, set yourself some goal, start a project with friends … . What’s important is that you keep digging for a bit, even if it takes some effort. The good news is that most topics get more interesting, the deeper you dig. And of course you can adjust the direction of your dig (just don’t jump to the next hole at the first obstacle!). What you have to look for is your "positive feedback loop”: Then the more you learn, the more fun it will be and the more successful you will be, which will motivate you more and so on. It just has to come from yourself (teachers and professors can only nudge you in a direction, but the initiative must come from yourself).- Marein Rahn

Q: How does electricity produce from metals and a potato?



Q: What is the weakest build of an atom? Is it graphite?

Q: How do crystals begin to form, and how does it keep its solid state?

Q:What happens if a crystals was to become liquid? Or has that never happened?

Q: What academic advice do you have for someone who wants to pursue a career in software architecture? 

A: The field of Computer Science is incredibly broad, including circuit design, programming languages, Artificial Intelligence, Algorithms, Networking, etc.  You will probably want to get a degree in Software Engineering, rather than Computer Science.  But you don’t have to decide right away.  During your first two years in college (freshman and sophomore) you should take broad range of courses to see which ones really are interesting for you.  You cannot take too many Math courses!  Mathematics, and that kind of logical thinking, suffuses every aspect of computing and also science!- Joan Lucas

Q:What advice would you give your younger self if you could time travel?

A: My advice for someone going to college would be to ask any question that you have, right away (if you leave them for later,  they will just pile up). Don’t worry about what others think and be confident that every question you have is justified. -Marein Rahn

Q: What do zinc and cooper have in common to light up an LED light?

A: What zinc and copper have in common is that they are both metals, and metals conduct electricity very well. Because they are different metals, they react differently with the fluid in the potato, and this gives rise to a voltage difference between the two. See the video on batteries in Pascale's presentation on Wednesday:

https://youtu.be/9OVtk6G2TnQ   -Hubert


Q: I wonder what is something Marie always followed and told herself when choosing to keep pushing forward not only for herself, children but also her work and studies to benefit even after she passed?

A: Marein: It’s a difficult question, since Marie wasn’t the kind of person who wanted to share too much of her personal life with the public. Maybe her drive was so strong because she was able two align two goals in her life: On the one hand, her work gave her an immense personal satisfaction, and on the other hand she grew up with the humanitarian conviction that all of us, individually, have a duty to improve the world. I think few people get to combine their own goals and life philosophy in this way. Either because they never get the chance, or because their personal desires are not compatible with a fulfilling cause (e.g. if your only goal is to get rich, you won’t have much to keep you going if you fail). Also, I think most of us often deceive ourselves into evading challenges, mostly for irrational reasons. E.g. sometimes if you are “scared" of an exam, you don’t want to study for it. I think Marie’s had such a logical and scientific attitude that she just didn’t accept obstacles as easily.


Another amazing story I didn’t get to mention:


Marie really didn’t enjoy the public attention she received in her later years (after the Nobel prizes), not even that of the early feminists of the time. But the editor of the US women’s magazine The Delineator, Missy Meloney, was so insistent that she started a campaign to fulfil Marie’s greatest wish. In the 20s, radium was already industrially produced, but Marie couldn’t afford any for her research (one gram cost 100.000 $, which would be more than 1.3 m. $ today). Mrs. Meloney managed to raise this money from her readers. So Marie agreed to write the “The Story of my Life” for the delineator, for "the encouragement to the women of America”. Together with Irene and Eve, Marie then travelled to Washington D.C. and was handed one gram of radium in a lead-lined mahogany box from president W. G. Harding. I have uploaded the relevant issues of the Delineator here: https://drive.google.com/open?id=1pgNCUT8hZSRzyuvL2samX9_YjgGpzFMm.


Q: Ask Frances Chadwick: What is the most challenging part of your job? 

A: the span. Trying to get a communication just right for 13,000 people is hard. And you can’t please everyone. You’re trying to take into account all the stakeholders (often including external as well), all the reactions, often with a deadline or some other pressure. And working with the federal government – well, it’s very bureaucratic, so that can sometimes add frustrations.

QI Was also wondering about the statistics of females in medicine in relation to the data shown in both Anna'a brief presentation and Irene Qualter's data.

Q:Could there ever be a technological development that would suppress radiation?

A: Good question! We can only really “suppress” radioactivity by shielding it. Atoms are radioactive when their cores (nuclei) are too unstable to hold together. The cores consists of neutrons and protons, which are held together by nuclear forces. These work quite differently from other forces that you are familiar with, like graviational or magnetic ones. It’s very difficult to manipulate the world at that scale, and we will not be able to change the “half life” (= average stability) of an radioactive element. However, in science you can never say never! If you were able to "design your own nucleus” (e.g. by adding neutrons), you might indeed make an unstable atom stable. Indeed, theorists have predicted an “Island of Stability” in the (largely empty) map of possible nuclei, see https://en.wikipedia.org/wiki/Island_of_stability . It’s not clear if we will ever get there though!- Marein Rahn

Questions to Frances Chadwick

Q: What are so things over here at New Mexico that are different from were you're from?” 

A: The food! The people! And the weather! Seriously, this is a great place to live - I can imagine if you grew up here, you would take it for granted, but NM is unique, and you should always remember how lucky you are. Another big difference is the open space/lack of population. The UK is a very crowded country - traffic everywhere, houses everywhere - its hard to get away. There are things I miss too of course - family, and the sense of history. 

Q: “What was the hardest thing to adapt to?” 

A: I think when you are a first generation immigrant, you always feel a bit “torn” - I love NM and working/living here, but I do also feel some nostalgia for places I knew as a child. But having said that, I have always felt very welcome here - not treated as an outsider, and I very much appreciate that. 

Q: "Do you think it is important for young women to be involved in the sciences from early on?” 

A: I dont' know. What I do know is that at some point - Sometimes at a young age, other times later on, I think it helps to find something that genuinely interests you. Often it is a teacher who can create this spark of interest, or a parent, or it can be something you discover on your own. I have three kids - one decided what she wanted to do when she was a junior in high school, another doesn't know what he wants to do when he grows up but is studying a subject he is deeply passionate about and has been since he was about 10, and the third has absolutely no idea and still thinks all school is boring!! So i hold out hope that in time, the third will find something that does interest her because it makes life more enjoyable, for one thing!

Q: “What was the easiest thing to adapt to?” 

A: You can connect with people, no matter where they are from. I think humans are good at making those connections, if they try. 

Questions from 2020 Summer Camp

Monday June 8

Q:  Could people potentially go into the quantum realm and see something if     they stayed in there for long enough?A:  The quantum realm is not really a place you can go to. It refers to a set of conditions in which quantum effects become noticeable. Usually this refers to very small objects, like a few atoms, a system with particles of light,  or super low temperatures. However, we can measure and study these (usually small) systems and try to understand how the quantum bits and pieces are behaving.

Q:  What is Bragg's Law and how do you calculate it?A:  When x-ray waves hit a crystal, they can reflect off each atom in the crystal. Only in certain directions do all these waves line up 'in phase', meaning that wavetops from one wave line up with wavetops from the others. These directions depend on how the atoms in the crystal are spaced. As a result, the pattern of reflected x-rays reveals the structure of the  crystal. See  https://en.wikipedia.org/wiki/Bragg%27s_lawAlso, tomorrow's demo on diffraction patterns address that same problem, but at optical wavelengths, rather than x-ray wavelengths
Here is the derivation of Bragg's law: https://www.youtube.com/watch?v=FRDvRhCvuHg

Q:  How can you tell when an atom is radioactive? A:  By definition, radioactive atoms fall apart sooner or later. The 'half-life' of any particular radioactive substance is the time in which half of the atoms have decayed. For example, if you start with an ounce of Tritium, 12 years later only half of it is left, and 12 years after that, only 1/4 ounce of tritium is left. So if you observe your starting ounce for a while, you can tell you are losing Tritium steadily. However, if I have only one atom, you can't tell when it is going to decay. So if my tritium atom is still around after 50 years, can I say it is not radioactive? Not really. It might decay one day later.Here is a cool video that talks about radioactive decay: https://youtu.be/UYvx0O8itMAor here:  https://en.wikipedia.org/wiki/Radioactive_decayor here: https://chem.libretexts.org/Courses/Oregon_Institute_of_Technology/OIT%3A_CHE_201_-_General_Chemistry_I_(Anthony_and_Clark)/Unit_3%3A_Nuclei%2C_Ions%2C_and_Molecules/3.1%3A_Nuclear_Chemistry_and_Radioactive_Decay
In the lab, we can tell that a material or an object contains radioactive elements by measuring the radiation that comes out of them usually with an instrument called Geiger counter (https://en.wikipedia.org/wiki/Geiger_counter) . There are three types of radiation: alpha, beta and gamma. Alpha particles are particles formed by two protons and two neutrons kicked out of the nucleous.  (https://en.wikipedia.org/wiki/Alpha_decay). Alpha particles have a very limited ability to penetrate other materials and are stopped very easily bya. piece of paper. Beta particles are high-energy, high-speed electron or positron emitted by the radioactive decay of an atomic nucleus (https://en.wikipedia.org/wiki/Beta_decay). Beta particles  can penetrate a sheet of paper, but can easily be stopped by a thin sheet of either Perspex or aluminum. Gamma decay is the process by which the atoms desintegrate emmitting a gamma ray photon. (https://en.wikipedia.org/wiki/Gamma_ray#Radioactive_decay_.28gamma_decay.29)We are constantly showered by radiation called cosmic rays (https://www.youtube.com/watch?time_continue=5&v=_bKbMARsE-4&feature=emb_logo)
Did you know that several household items in your house contain radioactive sources? When you figure it out, let us know!
Brief story about Marie Curie: https://youtu.be/w6JFRi0Qm_s
Q:  "Exotic magnets" were mentioned earlier, what are they? A:  Here is a cool series of videos about magnetic materials and magnetism. There are 6 episodes: https://www.youtube.com/watch?v=62dez4tD5Ok&list=PLiAlc6pI18dWJT9NNRgGxJF6yHgEElwmU

Q:  Do we always have to show our faces in the camera?A: Yes, except when we are on breaks. Why? because they provide feedback to us about your level of engagement and understanding. Humans we rely on facial expressions as a feedback loop in relationships. We care and we want to know you are doing well. If for whatever reason you are not comfortable with it, please send me an email and we can talk about it allobet@lanl.gov

Q:  What happens to atoms that pass through a black hole? Do they become energy? What happens to atoms when a black hole "evaporates"?A: First, let's look at what a black hole is: when people picture a black hole, you usually draw a black ball. Now this black surface is not really a hard shell, like a black billiard ball, or a thin shell like a christmas ornament. It is a boundary in space (just like the boundary between NM and AZ) from which no light comes out, but there is nothing there in that place. When an atom falls into a black hole, nothing special happens when it falls through this boundary, it just keeps on going. But once it crosses the border, it will never come out.What happens on the inside of a black hole we really don't know, except that nothing of the atom is left except it's mass (= energy, because E=mc^2) The way a black hole evaporates is that it radiates like a warm object (getting hotter and hotter, until the final little speck explodes). The radiation is a form of energy, so the black hole slowly loses energy, which means it loses mass (again via E=mc^2). So will our poor atom come out? Not really, it was destroyed long ago, and what comes out in the end is an amount of energy equivalent to the mass of the atom. 
Q:  How does the phase change work between martensite and austinsite? Why are the particles arranged so differently but still form martensite?A:  Check it out here: https://www.youtube.com/watch?v=yR-6_lS9vtshttps://www.youtube.com/watch?v=wI-qAxKJoSU
Q:  What is Bragg's Law and what is it used for?A:  Check this out: https://www.youtube.com/watch?v=QHMzFUo0NL8
Q:  In shape memory alloys, what allows the atoms to rearrange and why can't other materials do the same?A:  In shape memory alloys, you have two states that are very close in energy. When you put some work by deforming them, you push the system in an energy state and when you add heat the whole system moves to that state and when cooled down it goes back to the lowest energy state.
Q:  In a crystal, How do you know if it is made of more than one component and how would you discover what components it is made out of?A: A way to determine the crystal structure of a material is using diffraction (https://www.youtube.com/watch?v=QHMzFUo0NL8). The diffraction pattern from a material allows us to see what atoms is it formed of and what is the atomic arrangement.
Q:  Why is a diamond the strongest materiel?A: Because of the rigit arrangement of the atoms and covalent bonds. check this our: https://www.youtube.com/watch?v=xJ5h9bwaaKk

Q:  How many women participated in the Manhattan Project, and what was the ratio of males to females that worked on the project?A: I do not know how many women participated because women had all sorts of roles at LANL and other sister laboratories during the Manhattan project. https://www.energy.gov/women-manhattan-projectHere is a 40 min symposium about the Women in the Manhattan project: from women who worked at Oak Ridge that worked in the Manhattan Projecthttps://www.c-span.org/video/?326373-1/discussion-women-manhattan-project

Tuesday June 9


Q:  Why did the magnets have to be touching the Battery to go through the coil train?A: Check out the explanation of how the magentic train works: https://www.youtube.com/watch?v=BWuICjRU3t0https://www.youtube.com/watch?v=9k7zywli4Vg

Q:  If you used a battery with a higher volt would it go through the magnetic train faster or would it just last longer?

Q:  How does the battery get hot when there is magnets on it?A: It is not the fact that there is a magnet. When current flows through a conductor, the conductor gets hot. That's why the incandescent lights emit light. They get so hot when you force current through them that they glow. The battery gets hot because you by making the magnets touch the copper wire when you use the electric train, a circuit is created and current flows without much resistence between the + of the battery to the - of the battery. In most uses of batteries, we don't draw su much current and batteries do not heat up much and that's why they last longer than during our experiments.

Q:  What is the most radioactive element known as of present day? A: Because it is a naturally-occurring element that releases a huge amount of energy, many sources cite polonium as the most radioactive element. Polonium is so radioactive it glows blue, which is caused by excitation of the gas particles by radiation. A single milligram of polonium emits as many alpha particles as 5 grams of radium. It decays to release energy at the rate of 140W/g. The decay rate is too high that it can raise the temperature of a half gram sample of polonium to over 500°C and subject you to a contact gamma-ray dose rate of 0.012 Gy/h, which is more than enough radiation to kill you.But it really depends by the definition of "most radioactive" for example: francium — its most stable isotope has half-life of 22 minutes. But precisely because francium is so radioactive, nobody ever made enough of it to even see. (If a solid lump of francium somehow came into existence, it would instantly vaporize from its own heat.)

Q: What makes a potato a good conductor? Is it the water or starch content?A: https://www.youtube.com/watch?v=GZiLVqGd9JY

Q:  How old do you have to be in order to have an internship at the lab?A:  https://www.lanl.gov/careers/career-options/student-internships/high-school/index.phpStudents who wish to participate in the High School Internship Program must meet the following requirements:-Have senior standing AND be at least 16 years of age at the time the internship begins.-Have and maintain a minimum cumulative GPA of 3.0 on a 4.0 scale-Attend a high school located in northern New Mexico-Pass a new employment drug text: The Laboratory requires successful applicants to complete a new employment drug test and maintains a substance abuse policy that includes random drug testing.
Q: What do each of the dials mean on the multimeter? I would like to learn more about voltage and what it means. A: Different multimiters have different readings possible. Typically they measure electric Voltage (V). Voltage is the difference in electric potential between two points. If volatge is 0, there will be no current flowing in a conductor. Example: take a copper wire and measure the voltage between the two ends, it should read 0V. But when you measure the voltage across a battery, you should measure a value different than 0 if the battery is not dead. AAA batteries shoudl read 1.5V when new.Another thing multimeters measure is electrical resistance which is often refered by the greek letter Omega (Ω.). Electrical resistence is how difficult for the electrons to flow through the material. Eg: if you take 2 pieces of a wire of different lengths, the electrical resistence will be proportional to the length of the wire. The electrical resistence depends on the type of material, the length of the material and the diameter of the material.Another thing a multimeter can measure if the current or electrical intensity that flows through a wire. It is measured in A (Ampere or Amp)Ohms law says that the intensity in a conductor is equal to the voltage across the conductor divided by the resistance of the conductor.Check it out! grab your resistor and conect it to a battery. Measure the Voltage at the battery first. Connect the leads of the resistor to the battery and measure the intensity flowing in the system. Now do the math and check if the resistance calculated is equal to the resistance in the label of the resistor!Multimeters There are two types of Voltage and Alternating Volatge (AC) and Direct voltage (DC). Batteries generate DC Voltage. Alternating voltage is when the voltage is delivered in sinusoidal form and it is often refered with a wigle on top of the sign for Voltage. The voltage received in households is AC at 60Hz so 60 wigles a second. [NEVER USE A VOLTMETER ON AN OUTLET OR ANYTHING PLUGGED TO THE WALL OUTLET]
https://en.wikipedia.org/wiki/Multimeterhttps://en.wikipedia.org/wiki/Ohm%27s_law
Q: When sphagettification occurs near a black hole, what causes the black hole to have more of a pull on your feet in comparison to your head?A by Kaisa: A: Gravity works in the sense that the farther away you get, the less it pulls. While standing on earth's surface, you have more gravitational force pulling on you than if you were to be in space, and black holes are just an extreme example of this. Because the gravitational pull from the black hole is so extreme, it just takes less distance for this effect to happen, resulting in spaghettification.
Q: Is there a device that can actively see and/or measure the magnetic field around an object?A: It depends on the strength of the field. The device is called a Gaussmeter or a magnetometer. I would typicall use a gaussmeter to measure the magnetic field around an object while I would use a magnetometer to measure the magnetic field created by a small sample. Gauss is one of the units of magnetic field measurement. It is characteristic of small fields while Tesla is another unit that represents larger fields https://en.wikipedia.org/wiki/Magnetometerhttps://en.wikipedia.org/wiki/Earth%27s_magnetic_fieldOne tesla (T) is equal to 104 gauss
Q: Regarding the issue about cooling the wires, why not test magnets in an already cool environment, or at least a controlled environment where we can put the room(?) into extremely cold temperatures?A: To avoid magnets from heating up, they are often submerged in liquid nitrogen and therefore at 77K. However, the larger the magnetic field one needs, the higher the amount of current needed and the more massive the coils are. If the coils are large, then there is a challenge of thermal transport between the inside heating up and the liquid nitrogen not being close enough to cool it down quickly. Remember resistence in a metal goes up as temperature raises so the more the coils heat it, the more resistent the coils are and the more resistent thet are the higher they heat up. To avoid this. scientists have optimized paths for the Liquid nitrogent (or coolant) to keep the coils as cold as possible througout the thickness of the coil.
This about this: if you want to cool a glass with water, how do you think it will cool faster: 1) if you put ice around the glass or 2) if you put ice around the glass and ice inside the liquid?
Q: What is the difference between a lowercase m and an uppercase M on a multi meter? A: small m typically stands for mili, or a thousand. of the unit. So 1 mV is a throusand of a Volt or 0.001 V. M typically stands for Mega. It stands for 1,000,000 of the unit. So 1 MV is 1000mmmVhttps://en.wikipedia.org/wiki/International_System_of_Units

Wednesday June 10

Q: Why is wavelength important in spectrophotometric measurements?A: One way to figure this out is to use small electrical probes in the  eyes or brains of say a shrimp and listen for electrical activity of  brain cells when you shine ligh of different colors. But there are easier ways. For example, we know that certain flowers have parts that reflect  or absorb UV light very strongly. We can't see these markings, but some insects can. So you make some fake flowers, some with these UV patterns, some without. To us they would look all identical. But then you notice  that the insects only seek out the ones with the UV markings. So you conclude that insects can tell the difference, and therefor can see UV.  BTW, human eyes have 3 different color sensors (red, green, blue), but  mantis shrimp I believe have 16. Some people have only 2, and are  colorblind, and apparently there are some people who have 4, and they   can distinguis colors that no one else can.Asia: Let us answer this question from a couple of angles. Wavelength is important in spectrophotometric measurements because:A spectroscopic measurement is, by construction, a measurement of radiation intensity as a function of wavelength. Hence wavelength is the independent variable in your spectroscopic measurement.Electrons in every atom have discrete energy levels, which result in a characteristic emission line pattern (i.e. as electrons cascade down they emit light associated with discrete energies, which translate to discrete wavelengths). A simplified graph of the Hydrogen atom emission/absorption spectrum in the range of visible light can be found here: https://www.thoughtco.com/definition-of-balmer-series-604381Not all measurements as a function of wavelength can be made with the same technology. Hence, for every wavelength range there is a unique detector designed to deal with this range. For example, space telescopes which collect data in X-rays and Optical/Ultraviolet (UV) simultaneously, require two separate detectors in sync with each other (e.g. XMM Newton X-ray detector https://www.cosmos.esa.int/web/xmm-newton/technical-details-epic and optical/UV monitor https://www.cosmos.esa.int/web/xmm-newton/technical-details-om). Different wavelength ranges also differ in how difficult it is to capture and measure the energy/wavelength of a given photon. For e.g. it is much technologically easier to measure a spectrum in the visible (optical) wavelengths than in the very high energy range like gamma rays.

Q: how do we know that other animals can see UV and other spectrums of light?A by Kaisa: A: Eyes are made up of rods and cones. Scientists have found that some species, such as birds or certain types of shrimp, have more cones than humans, allowing them to see more colors than humans. (Fun fact: magenta is the color that the human mind makes up because we cannot comprehend what the color actually looks like)
Q: Why can't we see other spectrums of light?A:  Normal human eyes have cells (in the retina, which is in the back of  your eye) that respond to colored light. One type reponds strongly to red,  another type to green, and the third to blue. If infrared light falls on  them, nothing happens. Same for UV. One reason why all creature's eyes are all sensitive in roughly the same region of the spectrum is that it is the color band where the Sun is the brightest. When you're designing an eyeball, you might as well choose to sit in the spectrum where there is the most light. If we had x-ray eyes, we would all be blind, since there is no x-ray light on the Earth's surface.
Q: what is the theory of star?A by Kaisa: A: Stars are initially formed by small particles (hydrogen; the lightest element) pulling on each other and when enough particles are present, the gravitational pull intensifies enough that the particles start to fuse. These reactions are what causes a star to create the energy to glow. What happens in the rest of the star cycle depends on the size of the star. If it is a smaller size, like that of our sun, eventually it will run out of hydrogen atoms to fuse and begin to become hotter and denser allowing helium atoms to fuse (the next lightest element). When it moves to fusing larger elements, it produces more energy, causing the star to get bigger and eventually so big the star explodes, leaving a white dwarf star in its wake. If the star is very large, the gravitational force will become so extreme that the star collapses in on itself, forming a black hole.
A: A star’s history begins whenever the local gas pressure (resulting from motion of gas particles – e.g. Hydrogen or Helium) is overcome by gravitational attraction of the gas particles (resulting from their mass). The gas then collapses into a small region and becomes packed so closely together that the compression causes first nuclear reactions to occur. That is the point at which the star ignites and is ‘born’. As time passes the star will keep on shining and gradually burn up its fuel. Once enough of it is burnt, depending on its mass, the star can swell up or explode, leaving a different form of star behind. This page explains different evolutionary paths of stars in more detail: https://astronomy.swin.edu.au/cosmos/S/Stellar+EvolutionAs correctly answered by Kaisa, the very first stars in the Universe were composed only from Hydrogen and Helium in the primordial (the way Universe started off initially) composition of roughly 25% Helium and 75% Hydrogen. As the stellar evolution progresses elements heavier than helium are formed in the stellar cores and in explosions, while the explosions deliver all the different elements into the surrounding space. As time progresses the enriched gas then collapses to become a new generation of stars and planets, including our own Earth. Interesting fact – the basic building block of life, Carbon, is the end result of Helium burning in the stellar cores. Without stars, there would be no Carbon or, in fact, other heavier elements. Therefore, one indeed can say that *we are made of Stardust*. [Asia]
Q: On an open field, how far can a laser beam (like the ones used today) be seen? Do the  wavelengths of the different colors affect the distance?A: This depends on the air quality, the power of the laser, and the color. Let's look at color first: it has to do with the same phenomenon that causes the blue sky. The sky is blue, even if the air is perfectly clean, because light is scattered randomly, depending on the color. The effect is called 'Rayleigh scattering', and it states that a light ray can travel unimpeded through air for a certain distance, and then it undergoes scattering, which means the light ray randomly changes directions, without changing color. Just how far depends on the color: blue light has a short scattering length, and red light has a much longer scattering length.  That's why the sky is blue: all that blue gets rattled around and comes at us from all directions, and the effect also makes sunsets red, when the light has to travel through much more air. So your first choice would be a red laser. Next is air quality. When choosing a color, you want to avoid wavelengths  that are strongly absorbed by all the other things in the air: water vapor, CO2, molecules of smog, dust etc. Some of these molecules can vibrate at particular frequencies (=colors), and you want to look those up and avoid them. 
Next, it matters *how* you are looking at the distant laser: suppose you arein a noisy crowd, and then a high-tone bell rings: you can hear the bell because none of the voices in the crowd produce such a high frequency - the little bell really stands out. If you made ear protectors that block out all low-frequency sounds, it would be completely quiet until the little bell rings. Similarly, if you know exactly the frequency (or wavelength, or color) of the laser you're looking for, and you get a filter that blocks all colors except for the one you want to see, you    have a much better chance of seeing your laser by.
Finally, if all else fails, get a more poweful laser. In the real world, lasers can go tens or hundreds of miles through air. For example, lasers can be used to communicate from the ground to  satellites, hundreds of miles up.
Q: When a star dies for how much longer will we still see it shining?A: It depends how far it is! The farther it is, the loger we will see it https://www.youtube.com/watch?v=ELD_JvSk6xU#action=share
Q: Why did the inventors of the blue LED recieve a Nobel Prize?A: " Although red and green LEDs had been around for many years, blue LEDs were a long-standing challenge for scientists in both academia and industry. Without them, the three colours could not be mixed to produce the white light we now see in LED-based computer and TV screens. Furthermore, the high-energy blue light could be used to excite phosphorus and directly produce white light - the basis of the next generation of light bulb.""What's fascinating is that a lot of big companies really tried to do this and they failed," he said. "But these guys persisted and they tried and tried again - and eventually they actually succeeded."https://www.bbc.com/news/science-environment-29518521
Q: Why is there multiple rainbows when I look through the glassA: Not clear what the situation is here. Was this when you looked through the spectroscope (the plastic tube with the slit on one end and the piece of CD on the other? Could you rephrase your question? (Hubert)
Q: Why is it called the electromagnetic spectrum? What does light have to do with electricity and magnetism? Can different light sources interfere with each other and change their wavelength?A: The answer to the first two questions are given elsewhere on 'Ask Anna'.  Here is the answer to the third question: yes, if you mix two light beams with slightly different frequencies, you can make a new frequency equal to the difference between the two incoming beams, but it is not easy. The reason is that the frequencies are so high (Here is a video where two beams get mixed where the difference between frequency is just 1 part in 6x10^15. (Hubeert)
   https://www.youtube.com/watch?v=CuGEPkI117s
   This video is a link from this discussion: https://physics.stackexchange.com/questions/56144/can-light-waves-cause-beats
Q: Do gases other than oxygen emit a light?A: Yes, you can make any atom emit light, gas or not. For example, the orange highway lights have bulbs with Sodium vapor, which emits strongly  in the orange. The older bright white street lights have bulbs that work with Mercury vapor. Neon lights of course use Neon gas. Some strobe lights use Xenon gas. Helium gas is special because it was first discovered in Sunlight, because of its unique spectrum of light. Only after that was Helium found on Earth. If you want to make a light with Iron, you can do that, but first you have to heat it up to where it melts and then   vaporizes - not easy, but it can be done. The Mars Rover, for example, has a powerful laser on board with which it can vaporize rocks, and then  a camera looks at the light coming from the vaporized rock, and by the spectral colors it can tell which atoms were vaporized.
Q: I remember using the convex lenses today, and how the view from outside our windows were flipped vertically on our index cards. I also remember hearing that our eyes see the world upside down, but our brain is what allows us to see things right side up. Is the shape of our eyes similar to the shape of the convex lenses?A: Yes, your eye uses convex lenses to make the upside-down image on your retina. The first one is right in front: if you close your eyes, and place your fingers on your eyelid, and then move your eyes left-right-left-right, you can feel that the front of your eye has a convex surface right in the center, about 1/2" diameter. This one is called the cornea, and it cannot change shape. The second one is behind the iris and pupil. This one can change its shape, becoming flatter or more rounded, this change happens when you change focus between looking at something in the distance (flatter), and something close up (rounder). The cornea plus the lens produce the sharp image, and allow you to focus on    close-by objects or distant things (but not at the same time). Q: How do we know that light is electromagnetic radiation?A: It's easy to tell that radio waves or microwaves are electromagnetic, because  we can see the batteries, power cord, copper wires and all that. For light, this is not so clear. But we know that atoms are made of electrically charged object (the protons in the nucleus and the electrons 'in orbit'), and if we hit them with a powerful electrical punch, light comes out. The inverse can also be done: hit material with the right kind of light, and electricity comes out. These things tell us that light and electromagnetism are part of the same phenomenon.
Q: I was just wondering about the stipend? A: The stipend for the 2020 Summer Phsyics Camp will be $300 when Materials are returned. If student decides to keep the cool Raspberry Pi and arduino, then stipend will be witheld. 
Q: How long after a meteor hits will there be shock waves for an average size?A:  The shock waves that travel through the earth (P-waves) travel 5-8 km/second, so if you are at 100 miles distant, the shock wave would reach you in about 25 seconds. The schock wave in the air travels at about  1/3 km/second, and would arrive about 8 minutes later. (Hubert)
Q: Can radioactive isotopes be used to treat other things besides cancer?A:  Cancer is the primary target for using radioisotopes in medicine. Radioisotopes are also explored for treating drug resistant bacteria and some viruses including HIV.  Radioisotopes are also used to sterilize equipment because it doesn’t leave any residue.A: Yes: radioactive iodine-131 is used to treat hyperthyroidism, when the thyroid produced too much thyroid hormone. But besides treatment, radioactive isotopes are used for diagnosing diseases. For example, Carbon-14 can be used to diagnose stomach ulcers caused by the bacterium h. pylori. Radioactive isotopes are often used to find tumors, before   treating them. (Hubert)
Q: How long does the radiation in each materials last? Also how intense over a period of time?A: The half-life, or the time for 50% of the radioisotope to decay, is usually relatively short for medical isotopes, since the goal is to have the decay occur on a similar timescale to the biological processes used for targeting.  For example, actinium-225 has a half-life of 10 days, which is well matched to the time for the peptide targeting to occur from the blood stream.  Rubidium-82, which we also make (via the parent strontium-82) has a half-life of 75 seconds.  This is used for cardiac imaging, and is just carried in the blood stream and gives an image of the blood flow and any blockages as the blood moves through.A:  This depends on the isotope. Each radioactive isotope has a *half-life*.    This is the time it takes for half of the atoms to decay. Halflives range from fractions of a second to illions of years and longer. In medicine, you don't want to choose one with a short halflife, because then you don't even have time to get it from the production lab to the hospital. Also, you would not want to have one with a long halflife,  since then your poor patient would be radioactive for months or longer. So the intensity depends on the halflife. If the halflife is one day, then the intensity of the radioactivity is cut in half after one day, and after 10 days, only 1/1000th of the activity is left. (Hubert)
Q: Are sensors installed in the manipulators to help with maneuvering objects?A: No sensors, you just have to practice!  The thickness of the windows makes this quite challenging.  For the chemists, they practice pouring with water to be sure they have the depth perception correct.  We also have new people remotely clean the hot cells, bag up waste, or set up equipment to get their skills refined….tasks that need to be done but where it is OK to make a mistake or take your time.  Working with a batch or product to go out can be rather stressful.
Q: What do you concentrate the isotopes with?A: During chemistry, we use pretty common techniques to concentrate materials.  We may gently boil a solution in the hot cell to evaporate off some of the solvent (water or buffer) and reduce the volume, which concentrates the isotopes left in the solution.  Most of our purification techniques rely on column chromatography, where the a solution is poured through a set of tiny beads with a ligand on the surface.  As the solution passes over the beads, the ions in solution interact with the surface of the beads.  Some will stick (what you want) and others will pass through (what you don’t want). This can be used to essentially concentrate the dissolve ions onto the column.  If you change the solution, you change the affinity of the ions…or you can then pull off your desired isotope into a smaller amount of solution. 
Q: Where were other parts of Cassini made from, like other labs or countries?A: Cassini/Huygens was a collaboration of teams from NASA, ESA (European Space Agency),  Italy, Austria, Germany, Spain, France, the United Kingdom, Norway, Finland and the United States.[Hubert]The other instruments were made at institutions from all over the US and even Europe.  Some examples:  The cameras that took the amazing pictures were made at the NASA Jet Propulsion Laboratory (JPL) in Pasadena, CA.  JPL is also the NASA center which oversaw the whole mission.  The magnetometer, a device used to measure the magnetic fields through which Cassini flew, was made in the UK at Mullard Space Science Laboratory (MSSL).  Mullard also made the instrument that detected electrons flying around in Saturn's magnetosphere.  The instruments designed to detect high-energy particles were made at Johns Hopkins University's Applied Physics Laboratory in Maryland.  And the radio antennae that detected lighting on Saturn were made at the University of Iowa.  Cassini, like most of NASA's space missions, was actually made up of instruments from all over the world.  So you don't actually have to be working at NASA to be a "NASA scientist". [Dan]
Q: What are some other camps or programs that are recommended so that we can further delve into the world of science?A: Every area and school has different opportunities, but I would suggest FIRST robotics as a way to learn more about robotics and I would check to see if there's any companies or government research organizations (like NASA) that take high school interns - some do! [Space Gal]

Q: Are women less likely to be considered for shows that have to do with science?A: It depends. If you're working to be the host of a science show on a major network, yes. It is more difficult because the vast majority of their viewers are male and the Network execs assume that those viewers will only respond well to hosts who look like them. I would argue that bringing in more female hosts would allow them to capture an audience they haven't reached previously. I'm hoping that this changes little by little each year. [Space Gal]
Q: What does a physicist do and how can I start looking into a career?A: The range of things physicists do is vast. But you could divide it into three groups: working in industry, working in research, and working in  academia. There is overlap between these categories. Industrial physicists do research to improve processes, or to invent new ones.  Many physicists at universities do both reaseach and teaching, and many searchers (for example at the Lab) mentor students and postdocs. Here is a link to a booklet from the American Physical Society about careers in physics:   https://www.aps.org/careers/guidance/guide/index.cfm   And you can always contact me. (Hubert, a physicist)
Q: Where do Saturn's Rings come from?A: Now there's a great question, and the answer is: we don't know. Look at this article:  https://www.universetoday.com/107353/where-did-saturns-rings-come-from/  And I always tell my students "Ten or fifteen years from now, when you   are a young scientist and maybe the world's expert on the rings of Saturn, you should come back and tell me the answer". (Hubert)A great question!  Saturn's rings are made of water ice particles ranging in size from fractions of a millimeter to a few meters in size. The rings are only about 100 m thick!   That's about the length of a football field.  Nobody knows for sure how the rings formed.  Two leading theories are 1) a largish moon got too close to Saturn and got ripped apart by Saturn's gravity into fragments, or 2) The rings are leftovers from Saturn's original formation 5 billion years ago. [Dan]
Q: When we talked about galaxies, we saw that their blue colour indicated stars were forming and red colour indicated passive galaxies, or essentially the death of stars, is that correct?A: Almost! ‘Blue’ galaxies are blue in the visible spectrum because the galactic light is dominated by young, hot stars whose spectrum peaks in blue. The ‘red’ ones are red not because stars ‘die’ or explode. The galactic spectrum is essentially only composed of stellar light of old stars – they are much cooler than the young ones and their spectrum peaks in the red. The red galaxies are also devoid of gas which can fuel future star formation, hence there is negligible chance for them to get the blue light in the future. [Asia]An example of a blue galaxy: M51 https://www.nasa.gov/sites/default/files/thumbnails/image/m51-and-companion_0.jpgAn example of red galaxy: M87 https://upload.wikimedia.org/wikipedia/commons/0/07/Messier_87_Hubble_WikiSky.jpg Q: How does the color of a galaxy tie into a redshift and blueshift when objects are moving either towards or away from you? I'm not sure whether these two concepts are even connected, but in that case why are the colors blue and red so often seen?A: The colour of a galaxy is in part tied to its distance away from us. Practically all galaxies in our Universe move away (recede) from us in a cosmological sense. The further away in distance they are, the more quickly they are receding (the famous Hubble’s Law, discovered by Edwin Hubble and demonstrated in his plot:  https://www.pnas.org/content/pnas/101/1/8/F1.large.jpg). This means, that for far-enough galaxies the reddening of spectra from the expansion of the Universe is relevant, making a star-forming galaxy seem red overall. However for galaxies that are closer to us, like the ones we saw images of during the talk (redshift of 0.08) the observed face-value optical colour of the galaxy is determined by its composition (i.e. whether or not there is active star formation, stellar explosions, ionised gas, old and young stars).When we refer to a galactic colour in astrophysics, we also mean the ‘rest-frame’ colour of it, not the face-value observed one. This means that in the description of colour we already account for the reddening due to redshift. In the scientific literature, the colour is defined as a ratio of fluxes in two sections of a spectrum, hence describing a relative contribution of, say, red with respect to blue or green. The reason why the colours red and blue are seen so often is because galaxies tend to spend very long times in each evolutionary state, hence statistically there is a higher chance of finding a galaxy which is either blue or red than for e.g. in-between. When we look across cosmic epochs, the distribution of galaxy colour changes from blue-dominated (i.e. more blue objects than red) to red-dominated and for this reason we interpret the blue-to-red transition as an evolutionary sequence. Also, the definition of something being ‘blue’ and ‘red’ spans a broad range of actual ‘colour values’, so, if you will, our definitions are rather ‘loose’. [Asia]  Q: What are the "green" galaxies marked as "green" for? If blue means birth of stars and red no more star formation, do the "green" names just indicate neither of the two?A: Good job, you nailed it. The name ‘green’ comes from a definition drawn in a colour-magnitude diagram. Actually, a good blog entry answering this question and the one above is here: https://blog.galaxyzoo.org/2014/02/21/the-green-valley-is-a-red-herring/. Everything all the way up to the ‘quenching’ section is spot on, however newest research puts a couple of small question marks on the final part of the blog.  [Asia] Q: Because of the redshift, we know the universe is expanding, right? What gives us a redshift indicating the universe is expanding?Yes, the redshift tells us that objects are receding (moving away) from us. This is, however, in a cosmological sense. It means that the object is not moving away from us through space, but rather *the distance between the object and us is increasing* as the photon travels, due to the expansion of the Universe. One way of visualising it for yourself is taking a balloon, blowing it up to a small ball and drawing two dots A & B with a marker. Then draw a waveform representing a photon (a wave with, say, 3-4 crests and throughs). Then blow up the balloon a bit more and you will see how A and B get farther away from each other, although their position on the balloon surface does not change. These are your galaxy and yourself affected by the expansion of the balloon, 2D Universe. Now look at the photon – it got stretched, the wavelength increased but not because the wave crests were arriving at increased separations in time due to the motion of the far-away galaxy (like in the case of Doppler Effect). The wavelength increased because the 2D spacetime of the balloon Universe got stretched. Now, our Universe is a 3+1 dimensional version of the balloon, so it’s harder or near impossible to visualise how redshift works in our scenario. A neat explanation of what we are discussing is here: https://astronomy.swin.edu.au/cosmos/c/cosmological+redshiftSo now that we know what the cosmological redshift is, let me address how we found the expansion of the Universe. Thanks to multiple people like Edwin Hubble we were able to measure how far an object is from us and measure the redshift of emission lines, finding that the further away an object is, the more quickly it is receding from us. An indication that we are, indeed, in the balloon scenario.  [Asia] Q: Where do we find a redshift?We find the redshift by comparing the wavelength of emission/absorption lines observed for an object and their ‘rest-frame’, laboratory wavelengths. Each atom has a unique emission/absorption line pattern dictated by quantum mechanics, which we can calculate theoretically and measure in a laboratory. This pattern comprises of several lines, which is a template of lines spaced at particular wavelength intervals. We measure a spectrum of a distant object and overlay such template, shifting it around the spectrum until it matches the observation. Once we have found the match, we measure the difference between the laboratory wavelength values and the observed wavelength values to determine the redshift. [Asia]  Q: I'm wondering how exactly the electrons jumping levels relate to different colors of emitted light. Does the number of levels jumped down relate to wavelength and therefore relate to color emitted?Well done! Although, a couple of small clarifications. Firstly, the energy levels are not evenly spaced (so we do not count the ‘number of levels jumped down’). Secondly, the probability of an electron making a shift between two particular energy levels is determined by quantum mechanics. There are also some transitions dubbed as ‘forbidden’, i.e. the probability of an electron making a shift is very small. For this reason, electrons hang out in the higher energy state for a longer while (which we referred to as a meta-stable state in our discussion), before they finally fall down – that’s the reason why your phosphorescent paper was showing a glowing pattern which didn’t vanish immediately. [Asia]

Friday June 12

Q: Is the ocean salty like the one on Earth? Is it drinkable?I really like this question!  I assume you're asking about the ocean beneath the ice crust of Enceladus?  The ocean is probably not as salty as Earth's oceans, because our oceans are salty due to salts in eroded soil and rock carried by rivers into the oceans. The rivers themselves are formed by rain.  Since there is no rain or significant erosion going on underneath the ice on Enceladus, the water there is probably more like freshwater on Earth.  As to whether it would be drinkable by humans, that is an open question.  Even if it isn't salty, it could be fairly acidic, since there are thermal vents on the ocean floor of Enceladus bubbling hydrogen gas up through the water, which has the tendency to make the water acidic.   We don't know if it is too acidic to drink, though.  After all, Coca Cola is quite acidic (it will strip the enamel off your teeth if you drink too much!), but we can certainly drink that. [Dan]
Q: How did Heather Bottom get involved with Dragon Space mission and Space X? A: While I was in graduate school for Aerospace Engineering at Caltech (at the time, I had obtained a master's degree, passed qualification exams, and was working torwards my PhD), I started looking into full time work. I wanted to put my engineering skills to use in an applied environment (i.e. at an engineering company). And at the time, I wasn't interested in getting a PhD. Engineers can, many times, work in the industry without getting a PhD. I submitted my resume to some companies and got the job with SpaceX. The company originally wanted to hire me as a thermal engineer (given my experience in grad school), but in the end the System's Engineering job was a better fit for my interests and skills. I took a small risk because I also didn't fully know or understand what a Systems Engineer's job entailsd. The job was hard to turn down, especially given the mission goal to send US astronauts to the ISS. And from there, I worked for about 3 years with the Dragon Capsule and astronauts who are now at the ISS. [Heather Bottom]
Q: What do you recommend doing before you graduate that helped you in college or helped you prepare for college?A: I think one of the best things you can do to be ready for college is to be open to new ideas and new experiences.  Getting out of your comfort zone can be very challenging, but this can open up future opportunities that you can’t even imagine today.  Meet different people, ask questions, and don’t be afraid to try something different.  [Eva]Ask for help when you need it!  Building up a good support system and looking for people who can guide you will both build connections and help you to succeed.  [Ellen]Absolutely, great addition!  Everyone needs help sometimes.  And then don’t forget to help others when you are in a position to do so.  Together, we accomplish so much more. {Eva]
One thing to do to prepare for college would be to think seriously about the types of things you enjoy doing and the types of classes or subjects that interest you the most. Dive in deep and read all you can about those areas; learn all you can! Do all you can! There are so many resources out there today. Take on new projects that you enjoy doing... new, old, different. College will be busy and you won't have as much free time to enjoy slowly learning and discovering new things on your own time, so use the time now to do so. The more you learn and explore now, the more prepared you'll be. Even if the knowledge isnt directly applicable to the work in college, the experience will shape you in ways you cannot imagine. You'll bring something to the table that other students may not have thought about already.  [Heather Bottom]
Q: Would someone with less tumors still benefit or choose the medication that is currently being made with radiation, or would it be smarter to opt for surgical removal just because the cancer hasn't spread throughout the body as much? Is the Radiation too much for the brain to handel, as in this treatment is not an option for brain tumors?A: The Ac-225 is in very early clinical trial, so the use is very limited right now to patients who have exhausted other treatment options.  Surgical removal of a large mass tumor may be a better option for some patients; alpha emitters are not usually a good choice for larger tumors because that type of radiation doesn’t travel very far.  Treatment of brain  tumors is further complicated because many drugs (whether radioactive-based or more traditional) do not pass the blood-brain barrier, and so getting medication to the tumor site is very hard.  And as you suspect, the brain is very sensitive to side effects.  Selecting a treatment regimen is very individualized and I am certainly not the expert! We try to engage with people who know more about treatment so we can bring new tools to the table as options. [Eva]
Q: How much weight can tension hold before breaking (with the basic mesh we used today?
Q: What exactly is an arduino? A:   Matt will have another slide about this in Mondays presentation. Hopefully it will clear things up. An "Arduino" is actually a specific brand of microcontroller board. They were the first (and probably still the best) so people often refer to any generic board as an "Arduino". Your board that says ELEGOO UNO R3 is what we are referring to when we say "arduino" for this class. The Elegoo is actually a cheaper knock off of an official Arduino brand board but it interfaces with the software and sensor, motors, etc. just the same. [Matt]
Q: My internet cut out when Dennis was explaining turbulence, so what is it, and what does it do?
Q: If there are just 2 digits of bits, how do we know what they are supposed to represent?A: [Joan] Good question!  Yes, everything, everything, everything inside the computer is either 0 or 1 (or on/off, or true/false).  A transistor is either holding an electric charge, or it is not.  An electrical current is either running through the wire, or it is not.So, all the ordinary things we do in life (like writing words) needs to be represented using only zeroes and ones.  Sixty years ago, people got together and agreed to use the ASCII coding scheme, so that we would all understand each other.  In ASCII,the letter ‘A’ is represented by the 8 bits:                              01000001and the letter ‘B’ is represented by the 8 bits:                        01000010and the letter ‘a’ is represented by the 8 bits:                         01100001etc., etc. for all the symbols on your keyboard.There is no particular rhyme-or-reason to why we give a particular code to a particular symbol.  Any coding will work, as long as we all agree on it, so we can all understand each other.  (Actually, there are some reasons, but they are too low-level for us to think about here)You can google “ASCII table” to see all the ASCII codesBecause ASCII uses 8-bits per symbol, ASCII can only represent 2^8 = 2**8 = 256 symbols.More recently people have invented Unicode which uses 16 bits per symbol.  This allows 2^16 = 65,536 possible symbols.  Wow!  That is a lot.  But there are so many different languages in the world (Chinese, Arabic, Greek, etc.) that don’t use A, B, C, so we needed more symbols  thanASCII can give us. OK, that finishes PART 1 of the explanation!  Let us move on to PART 2 Computers do many amazing things, but they originally were used as number-crunchers (e.g., computing the vibration of a bridge during an earthquake.  Will the bridge fall down?)   So, we want to represent numbers inside a computing, and we only have the two symbols 0 and 1.   We could use ASCII, but there is a better way.In ordinary life we write down numbers using base-10 (or decimal) notation. For example, consider the following numbers:10                    which is ten100                  which is one hundred1000                which is one thousand.We could also write these as:0010                which is ten0100                which is one hundred1000                which is one thousand.By putting some leading zeros in front.  The zeros in front do not change the value.  We usually don’t do this, but we can. Our number system is positional.  The meaning of the vertical line (i.e., the symbol 1) depends on where it appears (i.e., in which position it appears).0010 is ten, because the symbol ‘1’ appears in the second-column-from-the-right-end.0100 is one hundred, because the symbol ‘1’ appears in the third-column-from-the-right-end.We talk about the positions as the “one’s place”, the “ten’s place”, the “hundred’s place”, etc.0362 is three-hundred and sixty-two0623 is six hundred and twenty-three.The meaning of the curly symbol ‘6’ (is it sixty, or six-hundred?) depends on where the ‘6’ appears. This positional notation is different from Roman numerals.  In Roman numeralsCCXXXVII is two-hundred and thirty-sevenXXIIII is twenty-fourThe meaning of the symbol ‘X’ is ten no matter where it appears in the written number.So now that we understand our positional number system (decimal, using base 10), we can do exactly the same thing in binary (i.e., base 2).In decimal we have a “one’s place”, a “ten’s place”, a “hundred’s place”,  a “thousand’s place”, etc.  These are the powers of 10:  1 = 10^0, 10 = 10^1, 100 = 10^2, 1000 = 10^3, etc.In binary we have a “one’s place”, a “two’s place”, a “four’s place”, a “eight’s place”, etc.  These are the powers of 2:  1 = 2^0, 2 = 2^1, 4 = 2^2, 8 = 2^3, etc.So the binary number  1101 is:One copy of 1000 (ie the value 8)One copy of 0100 (ie the value 4)No twoOne copy of 0001 (ie, the value 1)So the value of the binary number 1101 is equal to 8 + 4 + 1 = 13Any number written in normal decimal (base 10) notation can be converted into binary, and vice-versa.   If you pull up the Calculator on your computer, you probably get a very simple calculator, with add/sub/multiply/divide.  Click on the top left corner and it will let you choose other kinds of calculators.  A Scientific Calculator will have buttons for sine and cosine, and logarithms and square roots.  A Programmer Calculator will have buttons that let you move between decimal, binary and hexadecimal (base 16) notationQ: If the jar were to have no bottom, would the water come out of the mesh if tilted upside down?

Monday June 15

Q: John Sarrao, what school would you recommend, UNM or NM Tech? Which one would be better to have a degree in and work at LANL?A: Both UNM and NM Tech are good schools. LANL has many interactions and collaborations with both schools, and many graduates from Tech and UNM work at the Lab. I don’t think you can go wrong with either choice. They both have strong science and engineering departments. UNM is larger than NM Tech, as is Albuquerque compared to Socorro. So, if you’re sure about what you want to study and prefer a smaller campus with fewer students per faculty, NM Tech could be the answer. On the other hand, if you prefer a larger school with more options, then maybe it’s UNM. If possible, I’d recommend visiting both campuses, meeting some students who are studying what you think will be your major as well as some faculty, and seeing what feels right for you. [John Sarrao]
Q: Hannah Mohr, How long does it take to take a picture in space? Does it require a lot of money?A: t took several years of research to design the satellites and launch them into space before we could get a picture, but now that we have them in orbit we can get a picture in about a day. It costs quite a bit to build, launch, and operate your own satellite, but there are many universities these days that have programs where undergraduates can be a part of that process! I hope this helps! It's always great to hear when people are excited about space research. 
Q: How many arches are there in Arches national park?Q: There are more than 2000 arches in Arches National Park. When Arches first became a national park in 1971, there were only about 90 known arches. It then took 20 years of searching from park rangers and visitors to find, document, and name all of the arches hiding within the park. Since the last complete surveys in the early 1990s, many arches have likely collapsed or been created, so no one knows the exact number of arches today. Fun fact: an opening in the rock must be at least three feet long to be officially counted as an arch!
Q: How do we change the circuit in order to have multiple LEDs connected such that they also blink? If we did this would they all blink at the same time or would they all be slightly delayed? Great question! Read my answer to the following question. Lesson 4 in the manual is essentially hooking up 3 LEDs to the Arduino. You can code to have them all come on at once, or vary which light is on and also how bright it is. Another answer to your question is that you can hook up multiple LEDs. The negative end goes into the blue strip, the positive LED end and one end to a resistor go into the same column. Then the other end of the resistor goes into a different port. You then add code to define the LED port (we used int ledPin=13, for the first one) and add code to set that port HIGH and LOW.  [Matt]
Q: What are some other activities we can do with the arduino?Take a look at the manual that is one the CD that is in your kit, I also have the manual posted to http://chamisaelementary.com/robotics/Documents/Super%20Starter%20Kit%20for%20UNO.pdf . It is full of different projects for you to do. We did Lesson 3 on Monday. Lesson 4 is a good extension of our previous lesson and also teaches another important concept about "pulse width modulation". It is essentially a similar circuit but you hook up 3 LEDs and give them variable power rather than full on or off. The code is also on the chamisaelementary web site if you get stuck with that. But feel free to jump around the manual and choose whatever project you would like to do. All of the code is also on the CD, and about half of it is on the web site. [Matt]
Q: What are some materials that can be used for 3D printing and could we possibly have 3D printers at home one day? Is anyone working towards that?A: That’s a great question. The most common 3D printing filament for home use is called PLA (Polylactic Acid); it is a corn based biodegradable plastic that is capable of making highly detailed parts. One issue with PLA is that it is not very tough (yes that’s an actual engineering term, think it will not survive  being hit with a hammer or dropped too hard) and it will warp if left in a hot location (car in NM summer). When home makers need tougher or more heat resistant prints they typically turn to PETG (disposable water bottles are often made of this) or ABS (plastic that Legos are made from). There are positives and negatives to each of these materials as well. 3D printers at home are becoming more and more common. Very few people have the exact printers that I showed in the video at home because they all cost between $5,000-$50,000. Currently home printers range between ~$200 and ~$5000 which is still a lot of money, but it is more accessible than it used to be. [Graham]
Q: Why do you think that Geomorphology or Geology is one of the scientific fields that lack diversity and is staying behind? A: people aren't completely sure. While the number of geoscience PhDs earned by women has gone up greatly- in many subfields it's about 50% women, 50% men earning PhDs now- the geosciences are the least ethnically and racially diverse of all the STEM fields. Some people think that one of the reasons geology lacks diversity compared to other STEM fields is because there's a perception that geologists have to be outside all the time and do lots of fieldwork. While this used to be the case, maybe 100 years ago, modern geology includes much more math, computer and experimental work in addition to fieldwork. This misconception also hasn't been helped by the "classic geologists" (most of whom are white males) who perpetuate that stereotype in geology departments across the country, or by geology professors who don't take into account students' diverse experience levels with the outdoors and lead them on unpleasant or dangerous field trips. People have recently started looking into the economic and social barriers that prevent minorities from spending time outdoors. Hopefully we can both improve access to the outdoors for everyone, and help people better understand that doing geoscience isn't all about eating dirt and going on long, treacherous hikes! 
Q: What are some ways in which we can get into coding and computing? Who was the first female computer scientist?A: The idea of a computer traces back to about 1840.  Back then there were lots of machines (steam engines, looms for weaving) but a computer is fundamentally different from these because it is a programmable machine.  Instead of doing just one task, like weaving cloth, it can do any task that you program it to do.  Charles Babbage of Great Britain was the first to try to build such a machine.  His friend and colleague, Ada Lovelace, is called the “first programmer” because her letters contain actual programs for this machine.  Babbage was never able to actually build his machine, because the technology of that day was so much more primitive than now.  For example, there was no electricity. Ada Lovelace was remarkable not only in writing programs, but in realizing that the computer could process any information.  Babbage was thinking of the machine purely in terms of doing arithmetic (add, sub, mult).  But Ada said that the machine could process and transform music.  About 1960 a computer programming language called ADA was developed.  But it never was widely used.The modern computing age started with the construction of the ENIAC in 1943-1946.  Because this was war time, many of the programmers for ENIAC were women.  One particularly famous one was Grace Hopper.  I guess you could call her the first woman computer scientist.  She made a fantastic contribution to computing.  She invented the compiler.   Earlier we wrote some Python source code (i.e., Projectiles Activity).  All source code needs to be translated into zeros and ones in order for the computer to understand and execute the code.  This translation used to be done by people, by hand.  And extremely difficult it was too!   Grace Hopper invented a program, called a compiler, that will automatically do this translation.   Think about it….a compiler is a program, that reads in a program (perhaps a Python program) and outputs a program (the zeros and ones of the machine code).  A program manipulates a program!  This was truly a revolutionary idea.  The project Grace Hopper worked on was funded by the Navy.  She had a long career in the Navy, and a battleship was named after her in her honor!
Q: How do arches form:That's a great question and one that isn't so simple to answer. [Paul]
I like to think about arch formation as a three-step process:Start with lots of large sandstone cliffs that soak up water every time it rains - even in the desert, water is the key here!Water flows through the rock and dissolves the cement holding the grains of sand together.The wind blows away the loose uncemented grains of sand and gravity pulls down bigger blocks of rock to create a circular opening and an arch!More great information can be found from Arches National Park if you want to learn more:Why are there so many arches at Arches?Types of arches

Tuesday June 16

Q: How do put a carbon filament into a build? Like do you have to load the printer a different way?A: The printer has two nozzles, one nozzle runs filament just like in the other printer I showed; except that the filament in this printer is a nylon plastic that had chopped carbon fiber added to it at the factory. Even with this carbon fiber, the printer works in the same way. The second nozzle has a spool of carbon fiber running through it. The fiber is about the same size a fishing line. The printer melts the string of carbon fiber into the layer below it and has a blade to cut the fiber at the end of the line. [Graham] Q: Can you make metal objects using the 3D printer?A: Yes! We can make metal parts using 3D printing. The most common method of 3D printing with metal is called laser sintering or laser melting. The way it works is that the printer lays down a very thin layer of powdered metal a very powerful laser then traces over that metal to either weld or melt the small metal particles in the powder. The printer then spreads another thin layer of powder and the process repeats. This short video gives a pretty good overview of how laser sintering works (https://www.youtube.com/watch?v=yiUUZxp7bLQ&t=76s). Another method of 3D printing with metal is similar to FDM printing which is the type of 3D printing I showed in my video. In this type of printing, metal powder is combined with a wax/plastic binder and turned into filament. This filament is 3D printed using the same concepts as normal plastic printing. After the part is finished printing, it is placed in a solvent bath which removes most of the binding material from the print. The part is removed from the solvent bath and placed into a sintering furnace. The furnace heats the part up to ~1200 oC which burns off the rest of the binder and welds the small metal particles together.
Q: Is there a way people could program computers to learn how to teach other computers by training them?A: It is something that the AI community is interested in! We work on things like using machine learning methods to help train other machine learning methods, so it is an active area of research.[Katie]
Q: When training robots to do certain things at your job, are there different people doing different steps?  (ex. one that builds the robot, one that programs sensors, and someone who trains the robot)A: Yes, we're often working together in a team, where different people have different roles on the projects.  In the case of the robot, there were several people who worked on different aspects of the robot, including building the robot and sensory [Katie]
Q: What special classes in college should I take if I were to work with viruses like COVID-19?A:  I would say virology, immunology, molecular biology and biochemistry. [Karissa]
-Q: So would a virus problem, have rapid growth or rapid decreases, like exponentially? Would it look really pronounced and what conditions would you have to take into consideration when working on a problem like that?A: Yes, corona virus did have an exponential increase when it started in each outbreak area. This depends on how contagious the virus is and how easily it spreads. There is a number called R0. If one person has coronavirus, R0 is the average number of people that will catch corona virus from that one person. When R0> 1, then it tends to grow exponentially. When R0<1 (for example R0=0.5), it tends to die out. In some countries, it appears to have decreased exponentially . You can look and the growth and decay rates on the John Hopkins website: [Karissa]https://coronavirus.jhu.edu/map.html
-Q: Can DNA be used to predict and understand people's personalities and their hobbies?A: Yes, in principle DNA could be used to predict and understand people's personalities and their hobbies - especially if you understand the epigenetic marks on the DNA and you have data inside the brain in real time. However, the current technology and scientific understanding is not yet advanced enough to do this. Maybe you could be the first one to predict a personality! {Karissa]
Q: What kinds of things does a software engineer do?A: A software engineer can also be called a programmer. It depends on the industry I think. In video games I was titled "programmer", in academia and the electronics industry I've was titled "software engineer". In any case, those two job titles put together make a good description; an engineer that programs. Ultimately, I solve problems by programming. The girls were doing that in the Arduino class before my presentation.  One of the reason I went into programming is the diverse nature of the trade. Just about every technical industry involves programming in some aspect. As a programmer you can work on everything from spaceships and science to movies and video games. [Hawi]
A: Software engineers explore the structure and behavior of computer programs and computer systems.The Institute of Electrical and Electronics Engineers (IEEE), world's largest professional organization includes a Computer Society (a sub-organization) that publishes the Software engineering Body of Knowledge (SWEBOK). The SWEBOK includes the following subjects, any and all of which Software Engineers  should be familiar with:
Software RequirementsSoftware DesignSoftware ConstructionSoftware TestingSoftware MaintenanceSoftware Configuration ManagementSoftware Engineering ManagementSoftware Engineering ProcessSoftware Engineering Models and MethodsSoftware QualitySoftware Engineering Professional PracticeSoftware Engineering EconomicsComputing FoundationsMathematical FoundationsEngineering Foundations
Many of these areas involve creating and bringing to life abstract models of the subject area or writing code in programming languages and programming scripts.
Hope that helps... {Terri]
You can probably access this resource....https://ieeexplore.ieee.org/xpl/RecentIssue.jsp?punumber=32-Q: Why is that different functions of the soundboard such as playing music or turning on the LED lights take different amounts of energy?A:The circuit on the little board that plays music is made up of many electrical components, such as transistors, resistors, capacitors, memory cells, and finally a small speaker.  The part that makes the LED blink anso uses many components like that.. None of these are 100% efficient, and they all lose a little bit of energy when they are working. How much energy is lost also depends on how the circuit is designed. Some designs are more efficient than others.-Hubert
Q: What are some benefits and drawbacks of going to a local school, such as UNM or NM Tech. Ashlee: I went to a local school (I'm a NC native, so it wasn't one of the NM colleges). If the school is accredited (for example, for engineering, you want to make sure it is an ABET accredited program), then there are no major drawbacks to going to a local school. One thing I liked for going to a local school was proximity to my family. I was far enough to live on my own, but close enough to visit for a weekend if I wanted to. I currently am in grad school at an out-of-state private university and being further away does make it harder to visit. If you attend an in-state public school, the tuition is often cheaper than out-of-state schools or private schools. I think also the culture at a local school will be more familiar. This can be a pro and con, depending on what you are looking for.I have also attended a community college (2-year college, Associate degree), which has the potential for transfering to a 4-year (Bachelor degree) college. This is another local option, which I highly recommend. Community college tuition is often cheaper and the class sizes are smaller, so it becomes more personalized. Downside is that they will not grant a 4-year degree, but that is not necessary for all types of jobs and the credits can often transfer.
Tiffany: I actually choose to come from out of state to UNM because it was affordable (more affordable than staying with my own in-state options). At the end of the day, hard work, a good resume and good recommendations are more important than where you go to school. You do not have to go to a fancy (and typically very expensive) private school to get a good job, especially in science. One of the great things about UNM is that it is near Sandia National Labs, the Air Force Research Lab and Space Weather group and Los Alamos National Labs, and has active collaborations with all three. Being near these places makes it easier to get internships, and making friends with professors and advisors can help you get opportunities into those places. 
Isabella: I did not go to a local school in New Mexico, but I think I can provide some perspectives on it. If you grew up in New Mexico, I highly encourage you to branch out and experience college in a different state (if possible). College is usually only ~4 years, so it is a great opportunity to diversify your experiences and possibly see what other cultures are like. There may be some drawbacks though, such as being farther from family and having to pay more if you are out-of-state. Some schools offer scholarships though that can make the price the same as if you were in-state (for me, this was usually specific to the school, and depending on the situation, they may offer it to you without you needing to apply separately!). When I was applying to college, I received a scholarship from my local rotary club. Ask your high school academic counselor of any local opportunities for scholarships!  College is a huge period of growth and offers new/exciting challenges; take advantage of this time and make the most of it. :) 
I hope this all helps. Feel free to give out my email if the students would like to chat more. 
Patrick: I found going to New Mexico State University beneficial. Classes were sort of small, so student/professor ration was good.Some of my high school classmates also attended NMSU which made study groups a bit more familiar.The lottery scholarship is now a benefit to cover tuition…assuming criteria are met and sustained.I cannot cite a drawback. I would likely make the same decision today as I did decades ago.
Cassandra: There are financial benefits to studying locally because the tuition may be less and many students can still live at home or receive more support from family. Students who attend school locally can stay close to their friends and family. The drawback to staying local can be not gaining/developing as much independence and not making as much of an effort to meet new people and try new things. Compare the resources available at the colleges you are considering. Sometimes larger schools have more resources available.   
Paul: It's affordable and that's a huge benefit! The only reason I could move 2000 miles away to live in a new place for school was because of a generous scholarship.
Q: Which is the preferred way of connecting a circuit, series or parallel?Matt: I'm not sure what you mean by circuit, so I'm thinking of solar panels in my answer. Great question by the way. So the preferred method would be determined by the device(s) you are powering. Let's say each panel generates 1.5 volts (same as one AA battery), and let's say each panel can put out 2 amps (that's the actual electricity that runs through the device). If you hook them in parallel you would still only get 1.5 volts, but you would generate 4 amps of electricity. If you hook them in series you would get 2 amps at 3 volts. So if you have a device that needs more voltage (think of something that uses 4 batteries rather than 2) you would want to hook things up in series. If you have something that doesn't need so much voltage and needs more electricity (think of something that drains your batteries very fast) then you want to hook things up in parallel. Thanks for asking!
Q: Is there any advice that helped you guys go through college? Anything you would highly recommend (from classes to friends or any advice)?Natalie: Definitely look at local scholarships, which come out in a packet, at least in Los Alamos. Don’t overlook the small ones either, they helped pay for a new laptop and a desk for me. There are a lot for people going into STEM. And start on them early. The questions are pretty generic, so I recommend looking at past years questions and start this summer. College applications are pretty time consuming but do not burn out before scholarships. LANL typically has an information day for their scholarship, and do not miss it. They give out deadlines, how to apply, and advice. You do not have to be an intern to apply. Girl Scouts offers a lot, including a $20,000 scholarship to be a National Gold Award recipient, and they also have a website full of other scholarships. Hope this helps!
Ashlee: I would recommend trying different types of classes. It is okay to change majors midway through! Exploration is key - it never hurts to try. I tried on-campus research, industry internship, and interning at LANL - all of which had very different experiences. The more you try, the more you will figure out what you like and what you don't like. I also joined different clubs and that was both good experiences and a fun way to make friends. However, don't overwhelm yourself. I did a lot my first couple of years in college (heavy courseload, many clubs) that it was hard to enjoy things or really explore what I liked. I would make sure to save some time for yourself (it will be hard, but you'll find a good balance as you try things)
Tiffany:  When I first started I tried to just focus on classes and put a lot of stress on myself; I went to a college out of state, far from home, my parents couldn't afford to help me, and worked hard to get to a university with some sort of scholarship. I didn't want to 'mess' it up. I remember being really upset the first time I got a "B" and thought I'd have to work harder and that can put a lot of stress on oneself. However, even with some B's on my transcripts, I had good relationships with teachers who saw I worked hard, and obtained good recommendations, and those can mean a lot to get summer internships. 
I also think extracurricular activities show you're outgoing and motivated, and aren't highlighted enough that they're important on resumes, especially in undergraduate work. They're also a great way to make friends. I joined a climbing club at one point and I made lifelong friends there. It also helped me destress.
Summer internships are a great way to learn what you do and don't like. I did three (very) different ones during undergraduate and they really helped me find my path forward. Apply even if you don't think you'll get it; if you don't try you'll never know. I was always told you couldn't get into Sandia without knowing somebody, but one summer I just happened to have the skills somebody needed, and I got the posting. Most (science) jobs understand that you may not have a lot of 'skills'; hiring undergrads allows us to help you obtain skills, and helps groups give back to the community. This is part of where showing off those hobbies and extracurricular activities on your resume can come in handy
The last piece of advice is to balance your class load. One semester I saw a road to take out a ton of important classes that counted towards my major. I ended up with three (advanced) math classes, two physics classes and one astrophysics class. I don't think I had a single "filler". It was the worst semester of my life and by the end,  I was happy to just pass my classes that semester. Try to keep some 'basic' classes. They can be a lot of 'busy' work, but having a change of pace from your more intense major classes is really nice. Even a basic dance or workout class is a great way to get your mind off the harder work and destress.
Isabella:With regards to the first question, some advice that helped me go through college is go to office hours! It might seem kind of scary to go to a professor's office hours, but it is so, so helpful and gives you the chance to ask questions you could not ask during lecture. You can go to TA office hours as well. Additionally, make sure you are involved with your department; that can range from joining clubs to helping out on academic committees, or simply going to department events. This really helped me feel a sense of community and belonging, and it provided me with a lot of opportunities I would otherwise not have been exposed to. Stemming off of that, it's really important to find something non-STEM that you like to do, and force yourself to make time for it! It can be really easy to get overwhelmed by schoolwork and feel like you only have time for school. I promise you will be happier and more productive if you make time for hobbies and socialization. Prioritize your mental health! Hannah: I recommend making time for your hobbies and activities you enjoy. There will be times when school is like a firehose and you may have to focus all your time on it, but it is really helpful to have something non-academic to go back to. For me, that was symphony and taekwondo, which was where I made a lot of friends. It can help to make a nice meal every week or two, taking some time to take care of yourself. 
Something that took me a while to really grasp is that professors are there to help you, not just grade you. If you are having issues, go to office hours, send emails, ask for help. In my experience, most professors really want their students to be excited and engaged, so they are more than willing to spend the extra time. 
It is really helpful to keep your notes and assignments organized in case you need to refer back to them later. A lot of classes build on each other, and it can save a lot of time to have examples of what you learned a year ago. A few of my classes allowed a note sheet on the final, which was typically a single piece of paper - I ended up using those notesheets over and over again while reviewing for other classes because it was basically like a tiny summary of the class and all the important equations. If you decide to go to grad school, those notes will probably come in handy again.
I wish I had taken a Linear Algebra class earlier than I did (I took it as a senior, but I started needed the material as a junior). 
A final note: if you are struggling, remember you are not alone. Classes can be extremely difficult at times, and it is easy to feel isolated. Help your classmates, get their opinions (obviously only on assignments where it is allowed). I found having a study group really helped me make friends, inspired me to do the homework in a timely manner (so I would have a better idea what was going on when we studied), and helped me understand the material at a deeper level. 
Cassandra:  Take advantage of all the resources available to you in college! Interact with your professors and academic advisor. Get involved in organizations doing work you are passionate about. Use the resources available through the Career Center, Tutoring Center, and Library, and any other student support offices. In addition to getting excellent resources, you will also have the opportunity to network and meet amazing people who can support you throughout your studies and beyond. Consider studying abroad for a semester. In many cases the credits you earn will fulfill degree requirements. I also highly recommend learning a foreign language. Take the time to learn how to navigate your college's academic policies and processes (e.g. registration dates, drop dates, withdraw dates, course repeat policies, payment and refund schedules, etc.). Learn your degree requirements. Do not solely rely on someone else to pick your classes or make sure you are meeting your degree requirements. Appreciate all of your courses, even those that are not directly related to your major. Be strategic and select non-major courses that will complement your major. It is important to be well-rounded and develop transferable skills.Visit your professors during their office hours to discuss the things you are learning in class and ask questions. If you need help in a class, get itimmediately and do not wait. Consider a double major or minor if you are having trouble deciding between two degrees. If you decide early enough, it is usually possible to do both without adding to the amount of time you are in school.Enjoy the journey! It is important to be proactive and involved but also find balance so you can be present in the experience and not be too stressed to enjoy your time in college. Patrick: Be wise when selecting you class load. Signing up for 18+ in a semester is significant.Realize college in like a job. It is up to you to have the discipline to attend class and do the work. It’s unlikely you will have someone reminding you to get up and around for class, do your homework, study. College is a challenge, but don’t get frustrated, stick with it and achieve your goal.Take advantage of Professor office hours; usually posted on, or near their office door (find that location). Don’t be afraid to go to them to ask questions. A semester moves quickly and if you miss something or don’t understand something, it will pass quickly and the confusion can compound.Study groups are good; at the same time be self-reliant on learning the material…they won’t be taking the test for you.Think about residing on campus early in your college career. Getting accustomed to being on your own while still being a student will help by being in a dorm versus an apartment.
Paul: Get involved in activities and groups outside of your classes. Studying and classwork is important, but one of the best parts of college is getting to meet a whole world of new people. Plus it's always nice to have friends and activities that are separate from the classes and subjects you are thinking about all day - variety is the spice of life! Oh and don't be afraid to ask for help: study groups, office hours, and tutoring are not a sign of weakness!
Q: Is there any scholarships for high school students? Do you know reliable websites or places where high schoolers can apply?Ashlee: For high schoolers, there are many scholarships out there. The company that administers the SAT (CollegeBoard) has a scholarship database where you can search through. In addition, a big scholarship is QuestBridge, which is aimed for financially needy students. QuestBridge has a college prep program for high school juniors to help them through the application process. They also have a college scholarship for high school seniors to apply for. CollegeBoard Scholarship Search: https://pages.collegeboard.org/scholarship-opportunities-from-collegeboardQuestBridge: https://www.questbridge.org/Also, applying for financial aid (FAFSA) can lead to scholarships (sometimes called 'grants' - you do not have to pay these back!). However, they might also offer loans. You do not have to accept everything (you could just accept the scholarship/grant). Cassandra: High school seniors who attend high school in northern New Mexico can apply for the LANL Foundation Scholarship. 
Paul: I would first ask your high school counselor; they will know the most about local and regional scholarship opportunities that can be hard to find online. But there are also national organizations that might be able to help with scholarships or reduced tuition:American Association of University Women (AAUW): I have heard the local chapters often have scholarship opportunitiesWestern Undergraduate Exchange (WUE): reduced tuition if enrolled at certain universities and degree programs. Check out their College Savings FinderSociety for Advancement of Chicanos/Hispanics and Native Americans in Science (SACNAS): local chapters at Northern NM College and UNM
Q: To Patrick Garcia: did you ever feel overshadowed by your past relatives? If you did what did you do to "surpass" them?Patrick:Keep in mind the time in history were are discussing. My grandfathers were supporting the very early constructing of the laboratory and the United States were in the middle of World War II (WWII) in the 1940s. My family has many, many generations living in the Pojoaque Valley and at the time of WWII in general, a High School graduate was pretty-well educated. My grandfathers were each a heavy machine operator and carpenter. There is nothing wrong with either of those professions and the served a real need at that time. One grandfather had a HS diploma the other didn’t. Now on to my parents who both worked at the Laboratory; they both attended UNM. My mother graduated with an undergraduate degree in HS education and my father was drafted to the US Army and did not complete a degree. At the Laboratory my mother’s role was a Human Resources generalist and my father was responsible for a warehouse providing materials and components for experiments and testing to support the mission of the Laboratory during the Cold War. Based on their experiences and interactions at the Laboratory, they both instilled the importance of getting a college education. My older siblings and I all received college degrees (brother – Civil Engineering; sister – Finance) with mine being a BS-level degree in Mechanical Engineering. That degree enabled me to get the job I have and not be in a shadow of either my parents or grandfathers, simply because our roles and responsibilities were/are very different. At the Laboratory, I’ve advanced in position simply by doing the job I had at the time to the best of my ability and looked for opportunities to advance when I was ready…having the experience and technical background and then applying! Make your own path and cast your own shadow. Those actions are within your control. What others have done or have received recognition can be a metric, but is not the be-all, end-all. Success of an individual’s career is measured by the individual…are you satisfied you did all you could do when you look in the mirror. I encourage you to work hard now in school to learn and become technically knowledgeable, so you don’t have to work hard physically in the future (hope that makes sense). I’m happy to continue discussion or clarify my response if needed.

Thursday June 17

Q: Can you force the genes to turn on/off purposely to produce certain traits?A: Yes, you can use the new technique (invented by women, including my colleague Jennifer Doudna) Crispr/Cas-9 to add or remove genes to alter certain traits. [Karissa]
Q: How do you transfer the knowledge on your computer onto a robot such as the Roomba?A: It depends on the robot! Often the robot has specialized computer hardware so that it can run more efficiently.  We usually train a machine learning model on our traditional computers and then transfer it to the specialized hardware on the robot.[Katie]
Q: Can you store the power that you get from the solar panels?
Q: Is nuclear fission the best source of renewable energy?A: I won't want to say it is the "best", as we are still in the research development stage. We don't even know it will work or not for real, therefore, it is too soon to claim it is the best renewable energy. [Ning]
Q: Ning said she studied how Aluminium could be linked to Alzheimers, what results did she find?Although elevated level of aluminum was detected in some patients who were diagnosed with Alzeirmer's diseases, it is definitely not the only factor that caused this brain dysfunction problem. In addition, how does the aluminum get into the brain is not very clear. Regardless, some regions of the world has considered to lower the aluminum level in their drinking water standard. I have attached a link of a summary paper published a couple of years after the study we conducted at the medical School at the University of Kentucky. https://onlinelibrary.wiley.com/doi/pdf/10.1002/env.3170060303 [Ning] 
Q: Is there another isotope that can be used to power the rover?A:  Yes, both Americium-241 and Polonium-210 can be and have been used in the deep space explore mission. But plutonium-238 is the only one used to power the Mars rover. [Ning]
Q: How do you detect solar storms and why can you only detect it when it is already so close to Earth?
Q: What kind of tests do you do to test for impurities in the plutonium?
Q: What is the difference between UNM and NMSU?
Q: Is it possible to do two years at CNM then go to NM Tech?
Q: Can you get scholarships for being an intern?
Q: Does LANL offer internships for anything medical related?
Q: What lead Megan to choose Perdue besides the fact that it was far from California (in terms of academics)?
Q: What about international colleges? Do you recommend them? 

ASK ANNA Questions from 2019 Summer Camp

-Question: Although Global warming is happening, why are some areas in the world getting colder?

https://www.forbes.com/sites/startswithabang/2019/01/30/this-is-why-global-warming-is-responsible-for-freezing-temperatures-across-the-usa/#34272b0ed8cf

“When the vortex at the north pole becomes extremely weak, the high pressure zones found in the middle latitudes of Earth (where the westerlies are) can push towards the poles, displacing the cold air. This causes the polar vortex to move farther south. In addition, the jet stream buckles, and deviates towards more populous, southern latitudes. As the cold, dry air from the poles comes in contact with the warm, moist air of the mid-latitudes, you get a dramatic weather change that we conventionally refer to as a cold snap.

https://www.nationalgeographic.com/environment/2019/01/climate-change-colder-winters-global-warming-polar-vortex/

“First, it's important to understand the difference between climate and weather. Climate is defined as the average weather patterns in a region over a long period of time.”

“Scientists believe Earth will experience more extreme, disastrous weather as the effects of climate change play out.”

“As more Arctic air flows into southern regions, North America can expect to see harsher winters. That was the conclusion of a study published in 2017 in the journal Nature Geoscience. It found a link between warmer Arctic temperatures and colder North American winters. A separate study published in March of last year in the journal Nature Communications found the same link but predicted the northeastern portion of the U.S. would be particularly hard hit.”

https://www.nature.com/articles/ngeo2986#f4

“Warming temperatures in the Northern Hemisphere have enhanced terrestrial productivity. Despite the warming trend, North America has experienced more frequent and more intense cold weather events during winters and springs. These events have been linked to anomalous Arctic warming since 1990, and may affect terrestrial processes. Here we analyse multiple observation data sets and numerical model simulations to evaluate links between Arctic temperatures and primary productivity in North America. We find that positive springtime temperature anomalies in the Arctic have led to negative anomalies in gross primary productivity over most of North America during the last three decades, which amount to a net productivity decline of 0.31 PgC yr−1 across the continent. This decline is mainly explained by two factors: severe cold conditions in northern North America and lower precipitation in the South Central United States. In addition, United States crop-yield data reveal that during years experiencing anomalous warming in the Arctic, yields declined by approximately 1 to 4% on average, with individual states experiencing declines of up to 20%. We conclude that the strengthening of Arctic warming anomalies in the past decades has remotely reduced productivity over North America”

https://science.sciencemag.org/content/343/6172/729

“a rise in global mean temperature will almost certainly lead to an increase in the incidence of record high temperatures. Global warming also leads to increases in atmospheric water vapor, which increases the likelihood of heavier rainfall events that may cause flooding. Rising temperatures over land lead to increased evaporation, which renders crops more susceptible to drought. As the atmosphere and oceans warm, sea water expands and glaciers and ice sheets melt. In response, global sea-level rises, increasing the threat of coastal inundation during storms.”

 

https://agupubs.onlinelibrary.wiley.com/doi/full/10.1029/2012GL051000

“two hypothesized mechanisms by which Arctic amplification – enhanced Arctic warming relative to that in mid‐latitudes – may cause more persistent weather patterns in mid‐latitudes that can lead to extreme weather. One effect is a reduced poleward gradient in 1000‐500 hPa thicknesses, which weakens the zonal upper‐level flow. According to Rossby wave theory, a weaker flow slows the eastward wave progression and tends to follow a higher amplitude trajectory, resulting in slower moving circulation systems. More prolonged weather conditions enhance the probability for extreme weather due to drought, flooding, cold spells, and heat waves. The second effect is a northward elongation of ridge peaks in 500 hPa waves, which amplifies the flow trajectory and further exacerbates the increased probability of slow‐moving weather patterns. While Arctic amplification during autumn and winter is largely driven by sea‐ice loss and the subsequent transfer of additional energy from the ocean into the high‐latitude atmosphere, the increasing tendency for high‐amplitude patterns in summer is consistent with enhanced warming over high‐latitude land caused by earlier snow melt and drying of the soil.”

“We find that a warmer Arctic atmosphere contributes to dilated geopotential heights locally accompanied by lower heights across mid-latitudes and an equatorward-shifted jet stream. This allows Arctic airmasses to expand farther south while increasing the likelihood of heavy snowfalls. We find a distinction between early winter, when Arctic warming tends to affect only the lower troposphere, and mid-winter to late-winter when polar cap geopotential height anomalies is evident throughout the troposphere and lower stratosphere. When the entire Arctic atmospheric column is affected, the probability of severe winter weather in mid-latitudes increases, as observed during the era of AA in late winter. Colder Arctic conditions elicit the opposite response. These findings suggest that the continuation of rapid Arctic warming and melting contribute to more frequent episodes of severe winter across the Northern Hemisphere mid-latitude continents.” 

Dan Reisenfeld answered some of your questions:

1) Q: do tsunamis and earthquakes happen in space?”   I am taking it to mean do they happen on other planets or moons.  Of the planets and moons in our solar system, Earth is the only one with tsunamis, at least in recent geologic times.  Other planets have been measured to have small seismic activity, in particular Mars definitely has “Marsquakes”, although much smaller than on Earth.  In fact there is a mission on Mars right now called InSight, whose function is to look for Marsquakes.  A moon of Jupiter, Io, has active volcanoes (the only other place in the solar system to have them besides Earth), and so likely there are quakes on Io, too.  Venus could also have quakes, but we’ve never had instruments on Venus to measure them.  It is about the same size as the Earth and could very well have platectonics, like on Earth, which would give rise to quakes.

2)Q: What gives the planets their colors?  A: The reason depends on the planet. 

Here are a couple of videos that review the topics we worked on today:

Electricty and Magnetism

 https://www.youtube.com/watch?v=hFAOXdXZ5TM

https://www.youtube.com/watch?v=ru032Mfsfig 

 How does electricity work? http://scienceline.ucsb.edu/getkey.php?key=255

How did electricity start: http://scienceline.ucsb.edu/getkey.php?key=400

How does the atomic structure influence the PH?

https://study.com/academy/lesson/how-acid-base-structure-affect-ph-pk-values.html

 “Acids exist in an equilibrium with their conjugate base. The strength of the acid (pKa) depends on the stability of the base. When the proton leaves the acid, it leaves behind its electrons. Those are super negative and there is a big negative charge on the conjugate base. 

You know that if there is a concentrated negative charge, the base is not very stable. However, if the charge can be spread out, then the base is more stable, which means we have a stronger acid.“

“the stronger acid has a more stable base. One thing that influences base stability is the size of the ion. A larger ion can accommodate a negative charge better. Imagine that the charge has more 'room' to spread out. 

Luckily, we can use periodic trends to predict the size of the ion. This means we can predict the more stable base and therefore, the stronger acid. Ion size increases as we go from top to bottom of a column on the periodic table. (Ion size also increases from right to left, but that does not influence base stability due to electronegativity, which we'll talk about in a moment). “

LANL Visit Questions:

 One of the girls asked the following question and I was wondering if you would like to address it: "understanding how a generator that big powers a magnet, like what's the different things it does in order to power"

The generator is very similar to the little generators we played with during the summer camp. It has three magnets that rotate inside a coil of wire. The moving magnets creates electricity in a coil of wire, which in turn powers the magnet.

We need the generator because the power grid can’t deliver as much energy as we need quickly enough.  So we gather energy off the power grid slowly for an hour and store it, and then deliver it to the magnet in a second.

We gather that energy by using a motor to slowly spin up the generator. Energy is stored in the rotation of its massive shaft. Then we turn around and deliver that energy in the form of electricity to the magnets in just a second. 

Our magnets use only a little bit of energy (the biggest magnets use as much as my house does in a day for each 1 second pulse from 0 to 100 Tesla and back to 0). But the power the magnets use (how fast energy is delivered) is as much as an entire city.